Women's Health EOR

Ace your homework & exams now with Quizwiz!

Which human papillomavirus types most frequently cause genital warts?

6 and 11

What are the greatest risk factors for developing placenta previa?

Multiparity, smoking, and increasing maternal age.

Question: The ductus arteriosus connects which two fetal structures?

The pulmonary artery and the descending aorta.

What is the earliest gestation age an intrauterine pregnancy may be detected by transvaginal ultrasound?

5.5-6 weeks.

What vaginal pH is associated with vaginal atrophy?

≥ 5

A 12-year-old girl presents to the clinic with her mother. Which of the following is accurate patient education regarding the vaccine that helps to prevent the most common sexually transmitted infection? A. 12-year-old patients starting the vaccine series require two doses B. The vaccine can clear current infections C. The vaccine changes the recommended approach to cervical cancer screening D. The vaccine has not been proven to reduce the risk of anogenital cancers

A. 12-year-old patients starting the vaccine series require two doses It is incorrect to state the vaccine can clear current infections (B). The HPV vaccine is not effective in clearing current infections, genital warts, or anogenital intraepithelial neoplasia. Receiving the vaccine does not change the recommended approach to cervical cancer screening (C). To say the vaccine has not been proven to reduce the risk of anogenital cancers (D) is incorrect. The HPV vaccination series reduces the risk of cervical cancer in women.

Which type of pelvis is characterized by convergent side walls of the pelvic midcavity, forward inclination of the sacrum, and a narrow subpubic arch of the pelvic outlet that will most likely lead to labor arrest? A. Android B. Anthropoid C. Gynecoid D. Platypelloid

A. Android Labor is divided into three stages. The first stage is characterized by the onset of uterine contraction (every 3-5 minutes for more than 1 hour) until complete cervical dilation at 10 cm. The first stage consists of the latent and active phases. The latent phase is characterized by gradual cervical dilation until approximately 5 cm. The active phase typically occurs when the cervix is dilated to 5-6 cm and is characterized by rapid cervical dilation (1.2 cm/hour for nulliparous women and 1.5 cm/hour for multiparous women). The second stage of labor occurs from cervical dilation until delivery of the fetus. The third stage of labor occurs after the delivery of the fetus and ends with the delivery of the placenta. An abnormally long latent phase or second stage of labor is known as prolonged labor, an abnormally long active phase is known as protraction, and a complete cessation of progress is known as labor arrest. The most common indications for primary intrapartum cesarean section are protraction and labor arrest. Common risk factors for labor arrest include hypocontractile uterine activity (< 3-4 contractions/10 minutes, contraction duration < 50 seconds, or contractions < 200 Montevideo units) and cephalopelvic disproportion caused by fetal malposition or malpresentation. In particular, transverse arrest may occur due to constraint of the maternal pelvis and is most common in the android pelvis or platypelloid pelvis. The android pelvis is characterized by a narrow forepelvis, convergent side walls of the pelvic midcavity, forward inclination of the sacrum, and a narrow subpubic arch of the pelvic outlet. Active phase arrest occurs during the first stage and is defined as no cervical change for ≥ 4 hours despite adequate contraction or no cervical change for ≥ 6 hours with inadequate contractions. Labor arrest in the second stage is defined by failure to deliver after 3 hours of pushing or total duration of 4 hours in a nulliparous woman or failure to deliver after 2 hours of pushing or a total duration of 3 hours in a multiparous woman. An anthropoid (B) pelvis is characterized by a narrow transverse diameter, wide anteroposterior diameter, divergent forepelvis, narrow side walls of the pelvic midcavity, and wide inclination of the sacrum. A gynecoid (C) pelvis is the most common pelvis type in females and is considered normal. It is characterized by a wide forepelvis, straight side walls of the pelvic midcavity, and a wide subpubic arch of the pelvic outlet. Anthropoid and gynecoid pelvis types are the least likely to be associated with labor arrest. A platypelloid (D) pelvis is also associated with increased risk of labor arrest. However, unlike the android pelvis, the platypelloid pelvis is characterized by a narrow anteroposterior diameter, straight forepelvis, wide side walls of the pelvic midcavity, narrow inclination of the sacrum, and a wide subpubic arch of the pelvic outlet.

A 26-year-old nulliparous woman presents with her husband to her gynecologist with concerns about family planning. She states they have been unsuccessfully attempting to conceive for 2 years. Which of the following is the most common cause of the patient's condition? A. Anovulation B. Decreased sperm count C. Endometriosis D. Spontaneous abortion

A. Anovulation Infertility Workup Suitable for couples with no conception after 12 months of unprotected intercourse and female patients > 35 years with no conception after 6 months Hormonal, metabolic causes: TSH, FSH, free testosterone, DHEAS Semen analysis Ovulation causes: midcycle progesterone, LH, basal body temperature Decreased ovarian reserve: day-3 FSH Anatomic causes: hysterosalpingography, hysteroscopy, laparoscopy, ultrasound Decreased sperm count (B), endometriosis (C), and spontaneous abortion (D) are less likely causes of infertility compared to anovulation.

A 24-year-old G1P1001 woman is preparing for hospital discharge after a spontaneous vaginal delivery with no complications. Which of the following is the most appropriate recommendation to give her regarding when she can return to sexual activity? A. As soon as the woman feels ready after a minimum of 2 weeks B. As soon as the woman feels ready after a minimum of 4 weeks C. As soon as the woman feels ready after a minimum of 6 weeks D. As soon as the woman feels ready after a minimum of 8 weeks

A. As soon as the woman feels ready after a minimum of 2 weeks A minimum of 4 weeks (B), a minimum of 6 weeks (C), and a minimum of 8 weeks (D) are all incorrect because intercourse can be resumed after a period of 2 weeks of abstinence following uncomplicated vaginal deliveries. However, many women choose not to resume intercourse until well after 2 weeks given significant discomfort associated with coitus.

A 19-year-old woman presents to the urgent care with symptoms of a urinary tract infection. A urine specimen is obtained, and microscopic examination reveals the finding shown in the image above. A pelvic examination is subsequently performed and reveals an inflamed cervix with punctuate hemorrhages and frothy green discharge. Which of the following should the patient be informed about regarding her treatment? A. Avoid alcohol B. Night terrors C. Red-orange discoloration of urine D. Tendon rupture

A. Avoid alcohol Night terrors (B) and hallucinations are known side effects of oseltamivir administration for pediatric patients with influenza. Red-orange discoloration of urine (C) is associated with phenazopyridine, a urinary analgesic commonly used for relieving dysuria associated with urinary tract infections. Rifampin, a medication used to treat tuberculosis, can also cause red-orange discoloration of body fluids. Tendon rupture (D) is a side effect of fluoroquinolone antibiotics, particularly in patients > 60 years of age.

A 65-year-old woman presents to the clinic with right-sided abdominal fullness and early satiety that has developed insidiously over the past 2 months. Physical examination reveals right-sided adnexal fullness. Laboratory testing shows a cancer antigen 125 of 300 units/mL. Transvaginal ultrasound shows a right-sided adnexal mass with solid components and irregular septations. Which of the following is a risk factor for the most likely diagnosis? A. BRCA1 genetic mutation B. History of breastfeeding C. Oral contraceptive use D. Tubal ligation

A. BRCA1 genetic mutation Ovarian cancer is the second most common gynecologic malignancy. The most common histologic type of ovarian cancer is epithelial ovarian cancer. Ovarian cancer is most common in postmenopausal women, and the average age at the time of diagnosis is 63 years. Risk factors include Lynch syndrome, BRCA1 and BRCA2 genetic mutations, infertility, endometriosis, polycystic ovary syndrome, and cigarette smoking. The presentation of ovarian cancer varies and is often nonspecific. Patients may present acutely with bowel obstruction or pleural effusion caused by advanced ovarian cancer. Patients often present with a subacute presentation manifesting as pelvic pain or bloating, early satiety, abdominal distention, ascites, or urinary frequency and urgency. The diagnosis is initially considered in some patients when an adnexal mass is identified on routine physical examination or incidentally on imaging. The preferred imaging study to evaluate for an adnexal mass is a transvaginal ultrasound. The characteristic ultrasound features include a solid component within the mass, increased Doppler flow to the solid component, and irregular septations within the mass. Patients with symptoms or imaging findings concerning for ovarian cancer should also have a cancer antigen 125 level obtained to help assess the risk of ovarian cancer. Patients in whom there is high suspicion for ovarian cancer based on the history, exam, imaging findings, and tumor marker should undergo surgical removal of the adnexal mass and histologic evaluation to confirm the diagnosis. Surgery is required (rather than image-guided biopsy) because the mass must be removed intact to reduce the risk of spreading the cancer if the mass is found to be malignant. Many women who are undergoing surgery due to a suspicious adnexal mass will be diagnosed with benign etiologies. Patients diagnosed with ovarian cancer are typically treated with chemotherapy after surgery. Patients who are ultimately cured are monitored with cancer antigen 125 and routine assessment for new symptoms or exam findings. The prognosis for early-stage ovarian cancers is typically good, but advanced-stage cancers frequently recur. History of breastfeeding (B), oral contraceptive use (C), and tubal ligation (D) are each associated with a reduced risk of ovarian cancer.

A 28-year-old G2P1 woman presents to the clinic at 32 weeks gestation for a routine prenatal visit. She has an ultrasound performed to monitor a previously diagnosed placental condition, and it shows persistent homogeneous placental tissue extending over the internal cervical os. The patient reports no vaginal bleeding during her pregnancy. Which of the following is the most appropriate recommendation regarding delivery, assuming the underlying condition persists and the patient remains asymptomatic? A. Cesarean delivery at 37 weeks B. Cesarean delivery at 40 weeks C. Induction of vaginal delivery at 37 weeks D. Induction of vaginal delivery at 40 weeks

A. Cesarean delivery at 37 weeks Placenta Previa Patient will in third trimester Painless vaginal bleeding Diagnosis is made by ultrasound (transvaginal > transabdominal) Do not do a digital vaginal exam Cesarean delivery at 40 weeks (B) is incorrect. Placenta previa is an indication for cesarean delivery, but the delivery is recommended between 36-38 weeks gestation. Induction of vaginal delivery at 37 weeks (C) or 40 weeks (D) is incorrect because cesarean delivery is indicated for all patients with placenta previa.

A 25-year-old woman presents to the clinic for follow-up after testing was performed to evaluate her persistent vaginal bleeding and hemoptysis. She had a vaginal delivery of a term intrauterine pregnancy 4 months ago. Her hCG level is 100,000 mIU/mL. Transvaginal ultrasound shows an enlarged uterus without a fetal heartbeat. Chest X-ray shows multiple pulmonary nodules. Subsequent biopsy of the pulmonary nodules shows sheets of trophoblastic tissue consisting of syncytiotrophoblasts and cytotrophoblasts without villi. Which of the following is the most likely diagnosis? A. Choriocarcinoma B. Invasive mole C. Placental-site trophoblastic tumor D. Uncomplicated complete molar pregnancy

A. Choriocarcinoma Gestational trophoblastic disease occurs due to errors during conception, which lead to abnormal proliferation of trophoblastic (placental) tissue. Trophoblastic tissue normally leads to the formation of the placenta, thus gestational trophoblastic diseases are often considered diseases of placental tissue. The most common clinical manifestations are molar pregnancies, which can be complete or partial. Molar pregnancies are considered partial molar pregnancies when there is some fetal tissue. Complete molar pregnancies occur when an egg without genetic material (empty egg) is fertilized by a sperm. The genetic material then duplicates to produce a zygote with 46 paternal chromosomes. Complete molar pregnancies do not contain fetal tissue and have higher risk for malignant transformation than partial molar pregnancies. Partial molar pregnancies most often occur when an egg is fertilized by two sperm and produces abnormal cells with 69 chromosomes (46 paternal chromosomes and 23 maternal chromosomes) and possible fetal tissue. Partial molar pregnancies usually result in a miscarriage during the late first trimester or early second trimester and are less likely to become malignant. The histologic findings in partial molar pregnancies are trophoblastic proliferation consisting predominantly of cytotrophoblast cells (inner layer of the trophoblast) with focal hydropic (swollen) villi. The risk factors for gestational trophoblastic disease include extremes of gestational age (younger than 20 years or older than 40 years) and prior molar pregnancies. The symptoms and signs of molar pregnancy are often related to elevated hCG levels. Elevated hCG levels can cause nausea and vomiting, hyperthyroidism, and bilateral theca lutein cysts. Molar pregnancies may also cause abnormal uterine bleeding. Molar pregnancies are often suspected by the snowstorm appearance due to swollen villi seen on transvaginal ultrasound, but the definitive diagnosis requires a biopsy. Molar pregnancies are often treated with dilation and curettage and close monitoring to ensure hCG levels return to normal following treatment. Malignant transformation is an important complication of gestational trophoblastic disease. The types of malignant transformation, known as gestational trophoblastic neoplasia, include invasive mole, choriocarcinoma, placental-site trophoblastic tumor, and epithelioid trophoblastic tumor. Invasive mole is the most common type and is defined by invasion into the myometrium of the uterus. Choriocarcinoma is a rare and aggressive tumor that spreads hematogenously. Distant metastasis is common. The characteristic pathology finding is sheets of trophoblastic tissue consisting of syncytiotrophoblasts and cytotrophoblasts without villi. Choriocarcinoma may occur following a molar pregnancy, miscarriage or abortion, or following a preterm or term intrauterine pregnancy. Furthermore, choriocarcinoma can sometimes be part of germ cell tumors that are not related to gestation. Choriocarcinoma may present with abnormal uterine bleeding or bleeding from sites of metastasis, such as the lungs or gastrointestinal tract. The diagnosis is made with biopsy. The treatment consists of chemotherapy and sometimes surgery and radiation. Choriocarcinoma related to gestation has a good prognosis, but the prognosis is much worse in cases not related to gestation, such as those related to germ cell tumors. Invasive mole (B) is the most common type of gestational trophoblastic neoplasia. Invasive moles occur when the molar pregnancy invades the myometrium of the uterus. It usually occurs following a complete molar pregnancy. Biopsy showing trophoblastic cells and hydropic villi within the myometrium confirms the diagnosis. The patient in the vignette had a biopsy from a metastatic lesion that was consistent with choriocarcinoma. Placental-site trophoblastic tumor (C) is a rare form of gestational trophoblastic neoplasia. Placental-site trophoblastic tumors occur most often following intrauterine pregnancies, miscarriages, or abortions but can occur following a molar pregnancy. These tumors occur at the site where the placenta attaches to the lining of the uterus. Placental-site trophoblastic tumors rarely metastasize but can invade the uterus. These tumors are chemoresistant and are typically treated by surgical removal. The patient in this vignette had histologic findings consistent with choriocarcinoma and also had metastatic disease, which occurs more frequently in choriocarcinoma. Uncomplicated complete molar pregnancy (D) is incorrect because the patient has evidence of metastatic disease. Complete molar pregnancy without complications would be confined to the uterus. Furthermore, the expected histologic finding in complete molar pregnancies is trophoblastic cells consisting mostly of syncytiotrophoblasts with hydropic villi.

A 22-year-old woman is in labor, and you are watching the fetal heart rate monitor waveform. You notice the fetal heart rate gradually decreases at the start of the uterine contraction, reaches a nadir at the peak of the contraction, and returns to baseline as the contraction ends. Which of the following is the most likely cause of this change in the fetal heart rate? A. Compression of the fetal head B. Compression of the umbilical cord C. Fetal movement D. Uteroplacental insufficiency

A. Compression of the fetal head Compression of the fetal head causes a vagal reaction, which leads to an early deceleration of the fetal heart rate. Early fetal decelerations are often seen during the second stage of labor when the mother is actively using expulsive effort. The appearance of an early fetal deceleration on the fetal heart rate monitor waveform consists of a gradual deceleration in the fetal heart that starts at the same time as the uterine contraction begins. The nadir of the fetal heart rate deceleration matches the peak of the uterine contraction, and the deceleration returns to baseline as the uterine contraction ends. Early fetal decelerations are a normal and benign finding during labor, and they do not indicate an interruption in fetal oxygenation. Compression of the umbilical cord (B) causes variable decelerations. Variable decelerations have an acute fall in the fetal heart rate and a variable recovery phase. They do not have a clear relationship with the uterine contractions, which contrasts with early and late decelerations. Variable decelerations often occur in women with prelabor rupture of the membranes or decreased amniotic fluid volume. Fetal movement (C) often causes accelerations. Accelerations are an increase in the fetal heart rate ≥ 15 bpm lasting ≥ 15 seconds. They are reassuring. The most common causes include fetal movement, vaginal examinations, uterine contractions, umbilical vein compression, and fetal scalp stimulation. Uteroplacental insufficiency (D) often causes late decelerations. Late decelerations are defined by a gradual decrease in the fetal heart rate that begins after the start of the uterine contraction and reaches a nadir after the peak of the uterine contractions. Late decelerations are nonreassuring and can be caused by any decrease in uterine blood flow or placental disruption. Persistent late decelerations are an indication for cesarean delivery.

An 18-year-old woman presents to the clinic with recurrent pelvic pain each month with the onset of menses. Which of the following best describes the classic pain of the most likely diagnosis? A. Crampy and midline B. Crampy and unilateral C. Dull and midline D. Dull and unilateral

A. Crampy and midline Primary dysmenorrhea is a clinical diagnosis. Patients with a consistent history and exam do not require laboratory testing or imaging. However, the first-line imaging study to evaluate pelvic pain is a transvaginal ultrasound. Indications for a transvaginal ultrasound include unilateral pain or failure to improve after 3 months of nonsteroidal anti-inflammatory drugs, such as ibuprofen, or hormonal contraception. Patients should be educated that exercise and application of heat to the lower abdomen improve primary dysmenorrhea. The first-line pharmacologic treatments for primary dysmenorrhea are nonsteroidal anti-inflammatory drugs, hormonal contraceptives, or both. Combined (estrogen and progestin) oral contraceptives or progestin-only contraceptives are equally effective. Crampy and unilateral (B), dull and midline (C), and dull and unilateral (D) are incorrect because the pain associated with primary dysmenorrhea is classically crampy and midline. Primary dysmenorrhea is much more common than secondary dysmenorrhea. Unilateral pain suggests an alternative diagnosis. Although the pain may sometimes be described as dull, the most common description is a crampy and intermittently intense pain.

A 27-year-old woman who is positive for HIV presents to the clinic with bumps near her anus. On exam, there are skin-colored cauliflower lesions on the perianal skin. Which of the following is the best clinician-administered treatment? A. Cryotherapy B. Imiquimod C. Podophyllotoxin D. Rubber band ligation

A. Cryotherapy Condyloma Acuminata Patient presents with genital lesions PE will show cauliflower-like lesion Most commonly caused by HPV 6 & 11 Most common STI Imiquimod (B) is an immunomodulator that increases the immune reaction to condyloma acuminata. It is applied by the patient topically for a period of several weeks to treat genital warts. Podophyllotoxin (C) is an antimitotic drug made from plants. It is applied by the patient topically for several weeks. Rubber band ligation (D) is used in the treatment of internal (initiating above the dentate line) hemorrhoids. Internal hemorrhoids present most often with painless rectal bleeding with bowel movements. Internal hemorrhoids are graded according to the degree of prolapse. Patients with grade I-II internal hemorrhoids who do not respond to conservative treatment and patients with grade III internal hemorrhoids are treated with rubber band ligation.

Which of the following is routinely given to neonates at birth to prevent ophthalmia neonatorum in the United States? A. Erythromycin ophthalmic ointment B. Povidone-iodine solution C. Silver nitrate solution D. Tetracycline ophthalmic ointment

A. Erythromycin ophthalmic ointment Ophthalmia neonatorum, also known as neonatal conjunctivitis, is an acute infection that occurs within the first 4 weeks of life. Historically, this condition was commonly caused by Neisseria gonorrhoeae and was the primary cause of neonatal blindness before prophylaxis with certain medications became the standard of care. Nonsexually transmitted organisms can also cause this condition, such as Staphylococcus aureus, Pseudomonas species, Streptococcus pneumoniae, and certain viruses (e.g., herpes simplex virus). It is currently the standard of care for newborns to receive erythromycin ointment to both eyes within 2 hours of birth. Erythromycin is preferred due to its effectiveness against offending pathogens and its low side effect profile. Silver nitrate solution, tetracycline ointment, and povidone-iodine solution are also used around the world but are not commercially available in the United States. The main side effect of any of these prophylactic measures is chemical conjunctivitis, which typically resolves within 48 hours. Other routine procedures that are performed in newborns after birth include intramuscular administration of vitamin K1 to prevent vitamin K deficiency bleeding, hepatitis B vaccination, umbilical cord care to prevent infection, and monitoring for hyperbilirubinemia and hypoglycemia. Povidone-iodine solution (B), silver nitrate solution (C), and tetracycline ophthalmic ointment (D) are all effective against most offending organisms but are not commercially available in the United States.

A 30-year-old pregnant woman with a history of uncontrolled asthma presents to the obstetrics clinic at 30 weeks gestation for a routine prenatal visit. Her blood pressure measures 152/108 mm Hg and 154/106 mm Hg several hours later on recheck. She has no history of hypertension, and she reports no chest pain, shortness of breath, headache, vision changes, or abdominal pain. Laboratory studies include a urine dipstick negative for protein, 180,000 platelets/µL, creatinine of 0.7 mg/dL, aspartate aminotransferase of 25 U/L, and alanine aminotransferase of 28 U/L. Which of the following antihypertensives is the most appropriate choice to initiate in this patient? A. Extended-release nifedipine B. Labetalol C. Lisinopril D. Losartan

A. Extended-release nifedipine Gestational Hypertension Physical Exam: new-onset SBP ≥ 140 mm Hg or DBP ≥ 90 mm Hg at ≥ 20 weeks gestation If proteinuria or signs of end-organ damage develop: preeclampsia If BP elevation persists > 12 wks postpartum: chronic hypertension If BP returns to normal by 12 wks postpartum: transient hypertension of pregnancy Labetalol (B) is an alpha- and beta-adrenergic antagonist that is commonly used in the treatment of hypertension during pregnancy. However, it is contraindicated in this patient because she has uncontrolled asthma. Lisinopril (C) is an angiotensin-converting enzyme inhibitor, and losartan (D) is an angiotensin II receptor blocker. These medications are contraindicated in pregnancy due to the risk of fetal defects, particularly fetal kidney abnormalities with use in the second half of pregnancy.

A 42-year-old G3P2 woman presents in early labor. She is 39 weeks pregnant. Her first two pregnancies presented cephalically and were delivered vaginally at term without complication. One child was 8 pounds, 12 ounces and the other was 9 pounds, 7 ounces. Apgar scores at 1 and 5 minutes were 8 and 9 for both children. Her current pregnancy has progressed normally. Her glucose screening test performed at 26 weeks gestation was elevated at 151 mg/dL. Her 3-hour glucose test returned within normal limits. She is obese and has gained the appropriate amount of weight during this pregnancy. On physical exam, she is in slight discomfort. Contractions are occurring regularly every 5 minutes. She is 3 cm dilated and 100% effaced. The fetus is in frank breech presentation. Which aspect of the patient's history and physical exam increases the risk of a prolapsed umbilical cord? A. Frank breech presentation B. History of abnormal glucose screening test C. History of high birth weight infants D. Obesity

A. Frank breech presentation Umbilical Cord Prolapse History of malpresentation, PROM Cord precedes presenting part, increasing cord pressure and leading to fetal anoxia Treatment is emergent C-section If delay in C-section: Trendelenburg position, knee-chest position, bladder filling, elevation of presenting fetal part A history of abnormal glucose screening test (B) would not increase the risk of umbilical cord prolapse. In this case, the patient has a normal 3-hour glucose test performed. Therefore, gestational diabetes is unlikely even though her glucose screening test was elevated. History of high birth weight infants (C) would not increase the risk of umbilical cord prolapse. Risk factors for umbilical cord prolapse include low birth weight infants. Obesity (D) would not increase the risk of umbilical cord prolapse, although maternal obesity may complicate pregnancy and delivery in other ways.

A 35-year-old woman undergoes a transvaginal ultrasound showing hypoechoic, round, and well-circumscribed uterine tumors. Which of the following is the most common symptom of the most likely diagnosis? A. Heavy menses B. Infertility C. Intermenstrual bleeding D. Pelvic pain

A. Heavy menses Abnormal Uterine Bleeding Sx: heavy menstrual bleeding or intermenstrual bleeding Etiology Structural causes: polyp, adenomyosis, leiomyoma, malignancy or hyperplasia (PALM) Nonstructual causes: coagulopathy, ovulatory dysfunction, endometrial, iatrogenic, not yet classified (COEIN) Laboratory assessment: pregnancy test, CBC Endometrial tissue sampling for all women age ≥ 45 years First-line imaging study: transvaginal ultrasound Hormonal treatment is combination OCPs, IV estrogen, progestins, levonorgestrel IUD Infertility (B) predominantly occurs with fibroid tumors that distort the uterine cavity, such as subserosal or submucosal fibroids. Fibroids in these locations can result in difficulty conceiving and increase the risk of miscarriage. Intermenstrual bleeding (C) may occur in women with fibroids, but it is not the most common bleeding pattern. Furthermore, intermenstrual bleeding should cause consideration of endometrial pathology, such as endometrial hyperplasia or cancer. However, heavy menses can be a symptom of endometrial pathology as well. Pelvic pain (D) or discomfort occurs in women with fibroids but is less common than abnormal uterine bleeding. The pelvic pain or discomfort associated with fibroids is usually described as a chronic, intermittent, and dull pressure or pain.

A 50-year-old woman presents to the clinic for a routine well-woman exam. She reports it has been 12 months since her last menstrual period. Which of the following is the most common symptom of menopause? A. Hot flashes B. Mood symptoms C. Sleep disturbance D. Vaginal dryness

A. Hot flashes Menopause is the permanent cessation of menstrual periods and is defined in retrospect by a woman going more than 12 months without menses in the absence of other obvious physiologic or pathologic causes. The average age of menopause is 51 years old. Menopause before 40 years of age is considered to be primary ovarian insufficiency. The clinical manifestations related to menopause occur over a period of time. The perimenopausal period (menopausal transition) typically begins about 4 years before the final menstrual period and is marked by wide hormonal fluctuations leading to irregular menstrual cycles, hot flashes, sleep disturbances, mood symptoms, vaginal dryness, decreased bone density, and increased LDL. These symptoms generally continue into the first few years of menopause. Hot flashes are the most common symptom of menopause. Women report hot flashes occurring at night as night sweats, and they are often described as a sensation of heat centered on the upper chest and face and later becoming diffuse. The sensation lasts a few minutes and is often accompanied by significant perspiration. Hot flashes occur from once per hour to once per day but are most common at night. They often stop within a few years of the final menstrual period but may continue for up to 20 years. Bone loss begins during the menopausal transition, with most occurring in the year before the final menstrual period and the first 2 years afterward. Most women have a higher LDL following menopause. The diagnosis of menopause is established clinically. Follicle-stimulating hormone levels are often checked but offer no additional information. Many women do not require any treatment for menopause. Women who desire treatment for hot flashes may begin menopausal hormonal therapy. This may include unopposed estrogen in women who have had a hysterectomy or combined estrogen-progestin in women who have a uterus. The progestin is required in these patients to prevent endometrial hyperplasia. Hormonal therapy can improve hot flashes, sleep disturbances, depression and anxiety, and sometimes joint pains. Estrogen helps vaginal atrophy unless it is the sole symptom, then topical estrogen is the recommended treatment. Menopausal hormonal therapy is not recommended for women with a history of breast cancer, coronary heart disease, previous venous thromboembolic disease, or active liver disease. The standard duration of menopausal hormonal therapy is 3-5 years.

A 22-year-old woman presents with a complaint of painless heavy vaginal bleeding. She reports that her menstrual cycle occurs at irregular intervals and usually lasts more than 35 days. A physical exam is unremarkable at this time and laboratory studies are within normal limits. Which of the following is the most likely diagnosis? A. Menometrorrhagia B. Menorrhagia C. Metrorrhagia D. Polymenorrhea

A. Menometrorrhagia Abnormal Uterine Bleeding Sx: heavy menstrual bleeding or intermenstrual bleeding Structural causes: polyp, adenomyosis, leiomyoma, malignancy or hyperplasia (PALM) Nonstructual causes: coagulopathy, ovulatory dysfunction, endometrial, iatrogenic, not yet classified (COEIN) Laboratory assessment: pregnancy test, CBC Endometrial tissue sampling for all women age ≥ 45 years First-line imaging study: transvaginal ultrasound Hormonal treatment is combination OCPs, IV estrogen, progestins, levonorgestrel IUD Menorrhagia (B) refers to abnormally prolonged (> 7 days per cycle) or heavy (> 80 mL of blood) uterine bleeding that maintains a normal menstrual cycle. Causes include coagulopathy, endometriosis, leiomyoma, and neoplasm. Metrorrhagia (C) is abnormal uterine bleeding in between normal cycles that recur at irregular intervals. Causes of metrorrhagia include breakthrough bleeding, leiomyoma, endometriosis, or neoplasm. Polymenorrhea (D) refers to regular menstrual cycles that occur at shortened intervals (< 21 days). Causes include hormonal imbalance, stress, fibroids, and sexually transmitted infection.

A 22-year-old woman presents to the women's health clinic with genital sores and dysuria. Vital signs are T of 99.5°F, HR of 94 bpm, BP of 135/88 mm Hg, RR of 20/min, and oxygen saturation of 98% on room air. Which of the following additional findings on genital exam would be most suggestive of genital herpes simplex virus infection? A. Painful and fluid-filled vesicles B. Painless ulceration with raised and indurated margins C.Skin-colored cauliflower-shaped lesions D. Vaginal discharge

A. Painful and fluid-filled vesicles Genital Herpes Simplex Patient presents with a painful genital rash PE will show grouped erythematous shallow cluster of vesicles and lymphadenopathy Labs will show multinucleated giant cells on Tzanck smear (poor sensitivity) Diagnosis is made clinically, gold standard is tissue culture with polymerase chain reaction (PCR) Most commonly caused by herpes simplex virus (HSV) type 2 Treatment is acyclovir Pregnancy: acyclovir for 7 days after primary infection and from 36 weeks to delivery Painless ulceration with raised and indurated margins (B) is the classic ulceration seen in patients with primary syphilis who present with a chancre. Skin-colored cauliflower-shaped lesions (C) describes a classic presentation of condyloma acuminata, which is also called genital warts. Vaginal discharge (D) is a nonspecific finding that may be caused by gonorrhea, chlamydia, trichomoniasis, bacterial vaginosis, and vulvovaginal candidiasis. It is not associated with herpes simplex virus infection.

What is a theca lutein cyst?

An ovarian cyst that results from overstimulation by beta-human chorionic gonadotropin, such as occurs during molar pregnancy, multiple gestation, or clomiphene therapy.

What defines a reactive nonstress test?

At least two accelerations in a 20 minute period.

A 24-year-old G2P1 woman at 32 weeks gestation presents to her obstetrician for a nonstress test. Which of the following fetal heart rates is considered normal? A. 110-160 bpm with absent variability B. 110-160 bpm with moderate variability C. 170-220 bpm with absent variability D. 170-220 bpm with moderate variability

B. 110-160 bpm with moderate variability 110-160 bpm with absent variability (A) is incorrect because absent variability is not normal. Absent variability, particularly if associated with late or variable decelerations may cause fetal acidosis. Other causes of absent variability include sleep cycles > 20 minutes and exposures to medications such as analgesics, anesthetics, and magnesium sulfate. 170-220 bpm with absent variability (C), and 170-220 bpm with moderate variability (D) are incorrect because 110-160 bpm is considered the normal baseline fetal heart rate range.

23-year-old woman at 37 weeks gestation is being monitored in the labor and delivery unit after being admitted for persistent contractions and prelabor rupture of membranes. Her blood pressure is 118/78 mm Hg and her pulse is 88 bpm. A Doppler fetal monitor is used to determine the heart rate of the fetus. Which of the following fetal heart rates is within normal range? A. 109 bpm B. 142 bpm C. 161 bpm D. 175 bpm

B. 142 bpm Since 109 bpm (A) is below 110 bpm, it is considered fetal bradycardia. Both 161 bpm (C) and 175 bpm (D) are above the normal range and are abnormal findings that indicate fetal tachycardia.

A 24-year-old G1P0 woman presents to the clinic for her first prenatal exam. Based on her last menstrual period, she is 6 weeks pregnant. The patient asks when she will start feeling her baby move. At which of the following gestational ages is this patient most likely to start feeling fetal movements? A. 15 weeks B. 19 weeks C. 21 weeks D. 23 weeks

B. 19 weeks 15 weeks (A), 21 weeks (C), and 23 weeks (D) are not the best answer choices since fetal quickening typically starts between 18-20 weeks in women who have not had previous pregnancies, as is the case with the patient described in the vignette.

A 26-year-old G4P3 woman presents at 40 weeks gestation for induction of labor. She has a history of type 2 diabetes mellitus, and her body mass index is 37 kg/m2. During the second stage of labor, the fetal head delivers then retracts into the perineum. Gentle downward traction applied to the fetal head, while the patient's legs are elevated and her thighs brought to her abdomen, results in tearing of the perineum but is unsuccessful in delivering the infant. Maneuvers to deliver the posterior shoulder and placing the patient on her hands and knees are also unsuccessful. The obstetrician places the fetal head back into the pelvis. What is the best next step in the management of this patient? A. Applying more downward traction with simultaneous fundal pressure B. Cesarean section C. Clamping and cutting the umbilical cord D. Episiotomy

B. Cesarean section Applying more downward traction with simultaneous fundal pressure (A) is incorrect because it may result in injury to the brachial plexus, further shoulder impaction, or uterine rupture. Clamping and cutting the umbilical cord (C) should only be done after shoulder dystocia has been resolved if it is necessary to deliver the fetus. It should be avoided if the Zavanelli maneuver is performed in anticipation of a C-section because there will be no oxygen delivery to the fetus between the time the cord is cut and surgical delivery. Episiotomy (D) should not be done if the perineum is already torn during delivery because of the increased risk of extension of the incision, resulting in third- and fourth-degree tears. If the perineum is not already torn, episiotomy may facilitate delivery of the posterior shoulder but does not aid in release of the anterior shoulder.

A 23-year-old woman presents to her gynecologist due to fever, nausea, vomiting, and lower abdominal pain. She is currently sexually active and uses condoms occasionally. Vital signs are T of 38.4°C (101.1°F), BP of 138/78 mm Hg, HR of 90 bpm, RR of 18 breaths per minute, and oxygen saturation of 99% on room air. Physical examination is significant for bilateral lower abdominal tenderness to palpation. Pelvic examination is significant for purulent endocervical discharge, cervical motion tenderness, and uterine tenderness without adnexal masses. The patient is sent to the ED and admitted for treatment. Which of the following is a complication of the most likely diagnosis? A. Cervical cancer B. Chronic pelvic pain C. Endometriosis D. Ovarian torsion

B. Chronic pelvic pain Pelvic Inflammatory Disease (PID) History of multiple sexual partners or unprotected intercourse Lower abdominal pain, cervical motion tenderness, painful sexual intercourse PE will show mucopurulent cervical discharge Most commonly caused by Chlamydia trachomatis Outpatient treatment is ceftriaxone + doxycycline + metronidazole Fitz-Hugh-Curtis syndrome: perihepatitis + PID Cervical cancer (A) is almost always due to human papillomavirus (HPV) infection, particularly types 16 and 18. Risk factors for HPV-related cervical cancer include high-risk sexual behaviors, history of sexually transmitted infections, and immunosuppression. Risk factors for non-HPV-related cervical cancer include low socioeconomic status, increased duration of oral contraceptive use, and cigarette smoking. Endometriosis (C) is a condition in which endometrial tissue is found outside of the endometrial cavity, and it commonly causes dysmenorrhea, dyspareunia, and dyschezia. Risk factors include nulliparity, prolonged exposure to endogenous estrogen, late menopause, heavy menstrual bleeding, or obstruction of menstrual outflow. Ovarian torsion (D) occurs when the ovary rotates around the infundibulopelvic and utero-ovarian ligament. The most common risk factors for ovarian torsion are the presence of an ovarian cyst or neoplasm.

A 25-year-old G1P0 woman at 28 weeks gestation presents to her obstetrician for a routine visit. Her blood pressure was previously normal, until her 24-week visit when it was measured at 142/92 mm Hg. Today, her blood pressure is 144/92 mm Hg. She reports no vision changes, headache, or abdominal pain. Urinalysis is significant for 2+ protein. Complete blood count and kidney function testing is unremarkable. Which of the following is the most appropriate treatment at this time? A. Aliskiren B. Continued monitoring C. Labetalol D. Losartan

B. Continued monitoring Aliskiren (A) is a direct renin inhibitor and is contraindicated in pregnancy. Labetalol (C) is a first-line agent for the treatment of preeclampsia because of its rapid onset of action and safety profile. However, the patient does not need antihypertensive therapy at this point because her blood pressure is < 160/110 mm Hg and she does not have evidence of end-organ dysfunction. Losartan (D) is an angiotensin receptor blocker and, along with the angiotensin-converting enzyme inhibitor class, is contraindicated in pregnancy. Preeclampsia Pregnancy > 20 weeks gestation or postpartum Visual disturbances, severe headaches, or asymptomatic Evaluation will show new-onset hypertension (≥ 140/90 mm Hg) with either proteinuria (≥ 300 mg/24 hr or urine protein: creatinine ratio ≥ 0.3) OR significant end-organ dysfunction Treatment: delivery at 37 weeks (without severe features) and 34 weeks (with severe features) AND prevention of seizures with magnesium sulfate and prevention of permanent maternal organ damage New-onset hypertension < 20 weeks gestation: suspect molar pregnancy

A 25-year-old woman who is RhD negative presents to the clinic to establish prenatal care. She is 17 weeks pregnant with normal vital signs and positive fetal heart tones. Prior obstetric history is positive for three preterm births, each progressively earlier in gestation, with no living children. The father of her child is RhD positive (homozygous). A serum maternal anti-D titer is 1:32. Which of the following represents the best next step in the prenatal care of the patient and her fetus? A. Administration of anti-D immune globulin to the mother B. Doppler velocimetry of the fetal middle cerebral artery C. Fetal hemoglobin measurement via cordocentesis D. Serial intrauterine blood transfusions

B. Doppler velocimetry of the fetal middle cerebral artery Administration of anti-D immune globulin to the mother (A) is used as a preventative strategy during the first pregnancy of an RhD-incompatible mother and child. If autoantibodies have already been developed in previous pregnancies, plasmapheresis and intravenous administration of immunoglobulin G may be considered to slow the progression of fetal complications. Fetal hemoglobin measurement via cordocentesis (C) is performed after noninvasive methods such as Doppler ultrasound have suggested the presence of severe fetal anemia. The fetal hemoglobin as measured by cordocentesis will confirm the severity of the fetal anemia and guide decisions about intrauterine transfusion. Serial intrauterine blood transfusions (D) are administered to patients with RhD incompatibility who demonstrate severe fetal anemia. This includes fetal hematocrit under 30% or fetal hemoglobin more than two standard deviations below the mean value for gestational age. Rh Isoimmunization Rh-negative mothers exposed to Rh-positive blood → anti-Rh antibodies Subsequent pregnancies: jaundice, anemia, fetal hydrops, fetal death Anti-D globulin at 28 weeks (and within 72 hrs of delivery if infant is Rh+)

A 54-year-old woman presents due to vaginal discomfort. Symptoms began a few years ago and have become progressively worse. She reports daily discomfort and is no longer interested in sexual intercourse because of the pain. Her medical history includes seasonal allergies, for which she takes daily loratadine. Her past surgical history includes a tonsillectomy 10 years ago, partial hysterectomy 3 years ago, and a cholecystectomy 2 years ago. She reports no hot flashes, insomnia, mood swings, or difficulty with memory. A vaginal exam reveals sparse pubic hair, a narrowed vaginal introitus, atrophic vaginal rugae, and pale vaginal tissue. Which of the following is the most appropriate treatment option for this patient? A. Combination estradiol 1 mg and progesterone 100 mg oral tablet B. Estradiol 0.01% vaginal cream C. Estradiol 2 mg oral tablet D. Paroxetine 20 mg tablet

B. Estradiol 0.01% vaginal cream A combination estradiol 1 mg and progesterone 100 mg oral tablet (A) would not be the best therapeutic option for this patient. Since she reports only vaginal symptoms, a vaginal cream would be a better treatment option. Additionally, she does not have a uterus and thus does not require progesterone (which is a uterine-protective hormone added to estradiol to reduce the risks of unopposed estrogen). An estradiol 2 mg oral tablet (C) would not be the best therapeutic option for this patient. Since she reports only vaginal symptoms, a vaginal cream would be a better treatment option. Paroxetine 20 mg tablet (D) is a nonhormonal option for patients reporting vasomotor symptoms such as hot flashes or night sweats. It does not improve symptoms associated with vaginal atrophy.

A 25-year-old woman presents to the clinic for a routine wellness exam. She is considering contraceptive options. Which of the following contraceptive options has a black box warning due to the risk of blood clots? A. Copper intrauterine device B. Ethinyl estradiol and norelgestromin patch C. Etonogestrel implant D. Medroxyprogesterone acetate injection

B. Ethinyl estradiol and norelgestromin patch Copper intrauterine device (A) is a contraceptive device that is inserted into the uterus and provides contraception by creating an environment that is toxic to sperm and eggs. The copper intrauterine device does not release any estrogen or progesterone. It does not increase the risk of venous thromboembolism since it does not contain estrogen. Therefore, there is no black box warning related to an increased risk of venous thromboembolism. The benefits of the copper intrauterine device include its reversibility, high efficacy, avoidance of exogenous hormones, and long length of use. The copper intrauterine device can also be used for emergency contraception when inserted within 5 days of unprotected intercourse. The copper intrauterine device can remain in place for 10 years before it needs to be exchanged. The copper intrauterine device does not reduce menstrual bleeding, and it does not provide the same protection against pelvic inflammatory disease that the levonorgestrel intrauterine device provides. Etonogestrel implant (C) is a progestin-only implant used for contraception. Since there is no estrogen, it is not associated with an increased risk of venous thromboembolism, and thus there is no black box warning. The etonogestrel implant is a 40 mm plastic rod that is placed subdermally in the inner part of the upper arm. The etonogestrel implant is approved for 3 years of use but has been shown to be effective for up to 5 years. It is typically inserted under local anesthesia during an office visit. Women who have the etonogestrel implant inserted more than 5 days after the beginning of their last menstrual cycle should use a backup method of contraception for the first 7 days. The benefits to the etonogestrel implant include reversibility and high efficacy. Unscheduled bleeding is the most common side effect and reason for discontinuation. Medroxyprogesterone acetate injection (D) is a progestin-only injection. Therefore, it is not associated with an increased risk of venous thromboembolism, and there is no black box warning. The benefits of medroxyprogesterone acetate injections include high effectiveness, reversibility, and infrequent dosing (every 3 months). The contraindications of combined hormonal contraceptives related to estrogen use, such as prior venous thromboembolism, history of estrogen-dependent tumor, and liver disease, do not apply to medroxyprogesterone acetate injections. However, one possible disadvantage is that some women have delayed return of fertility after discontinuing medroxyprogesterone acetate injections.

A 60-year-old G6P6 woman presents to the gynecology clinic complaining of pelvic pressure and vaginal bulging. She also reports a sensation of incomplete emptying following bowel movements. Physical examination reveals prolapse at the posterior vaginal wall. Which of the following additional findings is most consistent with the most likely diagnosis? A. Complete urinary obstruction B. Fecal incontinence C. Stress urinary incontinence D. Urinary frequency

B. Fecal incontinence Complete urinary obstruction (A) is rarely caused by pelvic organ prolapse, but it is possible. It would most likely occur in association with anterior compartment prolapse, which is often caused by a cystocele (prolapsed bladder). The urethra can sometimes kink, which causes urinary tract obstruction. The patient in the vignette has posterior compartment prolapse. Stress urinary incontinence (C) is also associated with anterior compartment prolapse more than posterior compartment prolapse. Stress urinary incontinence occurs most often in women with mild prolapse and sometimes improves as prolapse progresses. Urinary frequency (D) is another urinary finding that may occur with pelvic organ prolapse. It is also more common in anterior compartment prolapse. Rectocele History of childbirth, trauma, previous surgeries PE will show a vaginal bulge at posterior vaginal wall or anterior rectum wall Most commonly caused by weak pelvic muscles Management includes managing constipation (high-fiber diet), pessary device, and surgery when conservative measures fail

A 35-year-old woman presents to her gynecologist with complaints of breast pain. The patient reports increased breast pain and swelling for 3 days prior to her menstrual cycle for the past several years. She became concerned when she noticed a mass in her right breast. Physician exam is positive for a small, fixed lesion in the upper outer quadrant of the right breast. Ultrasound shows a 1 cm lesion, and biopsy is recommended. Core needle biopsy reveals tissue nonproliferation with fibrotic changes. What is the most likely diagnosis for this patient? A. Breast cancer B. Fibrocystic disease C. Intraductal papilloma D. Radial scar

B. Fibrocystic disease Fibrocystic Breast Changes Risk factors: women 30−50 years old Sx: intermittent breast pain and tenderness that peak before each menstruation Ultrasound may show dense, prominent, fibroglandular tissue with cysts but no discernible mass Most commonly caused by fluctuating hormone levels during menstrual cycles Treatment is well-fitting supportive bras, applying heat to the breasts, or over-the-counter pain relievers Most common lesion of the breast Fibrocystic changes are generally benign and do not increase risk for breast cancer

A 31-year-old woman presents to her clinician. She and her partner are hoping to start a family soon, and she would like advice about having a successful and healthy pregnancy. Her family history is notable for asthma, and she has a personal history of hypothyroidism. Her partner has a child with spina bifida from a previous marriage. They are both otherwise healthy and make deliberate efforts to eat healthy and exercise. Which of the following recommendations should be made for supplementation during pregnancy for this patient, in addition to other recommendations for preconception health? A. Folic acid 0.4 mg daily B. Folic acid 4 mg daily C. Vitamin A 10,000 IU daily D. Vitamin A 5,000 IU daily

B. Folic acid 4 mg daily Folic acid 0.4 mg daily (A) is an appropriate daily dose to recommend for pregnant women to support organogenesis, but there is evidence that a higher dose leads to better outcomes for women at increased risk for fetuses with neural tube defects. Consumption in excess of vitamin A 10,000 IU daily (C) is known to be teratogenic in the first trimester. Vitamin A 5,000 IU daily (D) is the maximum supplementation dose recommended for women who are not from countries that are endemic for vitamin A deficiency, which can cause blindness, anemia, and increased risk of infection.

A 25-year-old pregnant woman presents to the obstetric clinic at 8 weeks gestation with dysuria for 2 days. She has had urinary frequency and urgency for the past 3 weeks. She reports no flank pain. Physical examination reveals no costovertebral angle tenderness. Urinalysis shows pyuria. Which of the following is the recommended treatment? A. Ciprofloxacin B. Fosfomycin C. Nitrofurantoin D. Supportive treatment with acetaminophen

B. Fosfomycin Cystitis Patient presents with low-grade fever, increased urinary frequency, dysuria, and suprapubic or abdominal pain Labs will show positive leukocyte esterase and nitrites Definitive diagnosis is made by urine culture Most commonly caused by Escherichia coli Treatment varies on age - adult, adolescents or pediatric Pregnancy: asymptomatic bacteriuria should be treatedComplications: ↑ risk of preterm birth, low birth weight, perinatal mortality Ciprofloxacin (A) is not recommended during pregnancy. Nitrofurantoin (C) should be avoided in the first trimester and at term if other options are available. It should be avoided in the first trimester due to mixed studies showing possible fetal birth defects, and it should be avoided within 30 days of term due to an increased risk of neonatal jaundice. Supportive treatment with acetaminophen (D) is incorrect because antibiotics are indicated in the treatment of cystitis. The patient has dysuria, and the urinalysis supports the diagnosis of cystitis.

A 27-year-old woman presents to the office due to new vaginal discharge. She has a long-standing history of recurrent vaginal yeast infections, for which she uses fluconazole 150 mg. She took two doses last week without relief. She reports having a thin, homogenous vaginal discharge that smells fishy. Which of the following organisms contributes to the suspected finding? A. Candida glabrata B. Gardnerella vaginalis C. Treponema pallidum D. Trichomonas vaginalis

B. Gardnerella vaginalis Bacterial vaginosis is the most common cause for vaginal discomfort and malodorous vaginal discharge. It is estimated to affect 30-50% of women. The pathophysiology of bacterial vaginosis involves Gardnerella vaginalis and a disruption in the normal microbiome of the vagina and decreased amounts of lactobacilli in the vagina. The microbiome then is replaced by Gardnerella vaginalis and causes a change in vaginal pH from a more acidic environment to a more basic environment. Gardnerella vaginalis and other pathogenic bacteria (including Prevotella and Mobiluncus species) thrive in a basic environment making treatment difficult. Risk factors for bacterial vaginosis include multiple sexual partners, risky sexual behavior, smoking, and douching regularly. It is less common in women using oral estrogen-containing contraception. Patients with bacterial vaginosis often present with a thin, white vaginal discharge with a foul or fishy odor. There are usually no other presenting symptoms, and most patients presenting with bacterial vaginosis are completely asymptomatic. Physical exam typically reveals a thin, milky discharge with a fishy odor. Bimanual exam usually has normal findings, and there is no cervical motion tenderness. A Gram stain will definitively diagnose bacterial vaginosis. If unavailable, Amsel criteria may be used in place of a Gram stain. Bacterial vaginosis can be diagnosed if three of the four Amsel criteria are satisfied: thin, homogeneous discharge; clue cells on microscopy or wet mount; a vaginal pH > 4.5; and a fishy, malodorous discharge with or without the whiff test. The whiff test refers to the olfactory evaluation of vaginal discharge when mixed with potassium hydroxide. The test is considered positive if a fishy odor is produced. Often, the odor may be appreciated without the addition of potassium hydroxide. First-line therapy for bacterial vaginosis is metronidazole 500 mg oral twice daily for 7 days. Vaginal preparations are also available, including 0.75% metronidazole gel intravaginally for 5 days. In pregnant women, bacterial vaginosis may be associated with spontaneous abortion and preterm delivery. Therefore, proper identification, diagnosis, and treatment are imperative. The risk of bacterial vaginosis is low, and the prognosis is typically very good. Unfortunately, patients with a history of bacterial vaginosis are more likely to have recurrent infections despite prevention. Up to 50% of patients will have a recurrence within 12 months. Candida glabrata (A) is a fungal organism that causes symptoms of a yeast infection. Patients typically present with a thick, white vaginal discharge that is often pruritic. Candida glabrata is a common cause of recurrent yeast infections due to the pathogen's resistance to azole antifungal therapy. Treponema pallidum (C) is a spirochete bacterium that causes syphilis. Symptoms of syphilis vary depending on the stage of disease. Genital symptoms usually include the development of a chancre, which is a painless, shallow ulcer that appears where the spirochete first entered the skin. Patients with syphilis do not often note a fishy vaginal discharge. Trichomonas vaginalis (D) is a sexually transmitted infection that produces a thin, frothy, yellow-green vaginal discharge that may be malodorous (but not typically a fishy odor). Trichomonas vaginalis may present with associated symptoms such as dysuria, dyspareunia, and pelvic pain.

A 63-year-old woman presents to the gynecology clinic for a routine wellness exam. Physical examination reveals a bulging of the apex of the vaginal vault. Which of the following is recommended to prevent this condition? A. Abstinence B. Kegel exercises C. Smoking cessation D. Weight lifting

B. Kegel exercises Uterine Prolapse Risk factors: multiparity, age, decreasing estrogen levels, trauma Rx: Kegel exercises, pessary, surgery Abstinence (A) is incorrect. Some women may have less desire for sexual intercourse as a result of pelvic organ prolapse, but abstaining from sexual intercourse does not prevent pelvic organ prolapse. Smoking cessation (C) has not been proven to reduce the risk of pelvic organ prolapse. Weight lifting (D) is associated with increased intra-abdominal pressure, which can exacerbate pelvic organ prolapse.

A 19-year-old woman presents with a tender genital ulcer that she noticed 2 days ago. She also reports dysuria and dyspareunia. On exam, you note a genital ulcer that has a ragged border and purulent base. A palpable, tender left inguinal lymph node is also present. Which of the following organisms is the most likely causative organism? A. Chlamydia trachomatis B. Haemophilus ducreyi C. Phthirus pubis D. Treponema pallidum

B. Haemophilus ducreyi Chancroid Risk factors: sexually active Sx: painful genital ulcers PE: papule evolves to a pustule which ulcerates, ulcers on an erythematous base covered by a gray or yellow purulent exudate and painful lymphadenopathy (bubo) Caused by Haemophilus ducreyi Tx: ceftriaxone 250 mg IM or azithromycin 1 g oral Chlamydia trachomatis (A) is an intracellular pathogen that causes the most common bacterial sexually transmitted infection (STI). Although the majority of cases are asymptomatic, patients may present with purulent urethral discharge, dysuria, postcoital bleeding, and low-grade fever. Phthrius pubis (C) is a pubic or crab louse that causes pediculosis pubis, a common STI that is transmitted through sexual contact or fomites (such as clothing, beddings, or towels). The hallmark clinical manifestation of pediculosis pubis is severe pruritus in the genital region. Treponema pallidum (D) is a spirochete that causes syphilis, an STI that has four stages of infection including primary, secondary, latent, and tertiary. Unlike chancroid, the ulcer of syphilis is nontender and has a raised, indurated margin with a nonexudative base. It usually resolves even without treatment. Because of this, patients who present with late-stage syphilis may not even remember having the ulcer in the first place.

A 34-year-old woman who is 8 weeks pregnant presents for a routine prenatal examination and reports persistent vomiting up to six times a day, and she has been unable to tolerate anything by mouth for the past 2 days. Her weight today is 114 pounds compared to 120 pounds 2 weeks ago. A blood test is ordered. Which of the following electrolyte imbalances is associated with the most likely diagnosis? A. Hyperchloremia B. Hypokalemia C. Hyponatremia D. Hypophosphatemia

B. Hypokalemia Hyperchloremia (A) occurs when there is a loss of fluid from sweating, excessive skin burns, severe dehydration, and diabetes insipidus but is not associated with excessive vomiting. Patients with hyperemesis gravidarum present with hypochloremic metabolic alkalosis. Hyponatremia (C) has a variety of causes such as diarrhea, diuresis, excessive sweating, drinking excessive water, and heart, liver, and kidney disease. It is associated with vomiting but is not typically found in patients with hyperemesis gravidarum. Hypophosphatemia (D) usually occurs as a result of kidney failure, hypoparathyroidism, diabetic ketoacidosis, or rhabdomyolysis and is not associated with hyperemesis gravidarum. Hyperemesis Gravidarum Peak incidence: weeks 8-12 Weight loss Hypokalemia Ketonemia Rx: IVF with 5% dextrose, antiemetics

A 32-year-old woman returns to her obstetrician for a follow-up of her glucose tolerance testing results and is advised her blood glucose levels were elevated with each of the tests. She has been trying to adjust her diet ever since her first prenatal visit and is frustrated that her glucose levels are elevated. She has obesity at baseline and a family history of diabetes mellitus. Her interim history and exam are otherwise unremarkable. Based on her test results, you discuss that medical management is recommended. Which of the following is the first-line pharmacologic therapy recommended for this patient? A. Glyburide B. Insulin C. Metformin D. Myoinositol

B. Insulin Gestational Diabetes Mellitus Screening between 24 and 28 weeks with 50 g glucose load (abnormal: glucose > 130-140 mg/dL after 1 hr) Diagnostic criteria A 100 g glucose challenge with > 95 mg/dL fasting, > 180 mg/dL at 1 hour, > 155 mg/dL at 2 hours, or > 140 mg/dL at 3 hours > 130-140 mg/dL after 1 hour challenge can be positive depending on facility and local prevalence Rx: lifestyle changes, fetal growth monitoring, insulin Fetal risks: macrosomia, respiratory distress syndrome, neonatal hypoglycemia ↑ maternal risk of type 2 DM Glyburide (A) is an oral option for the treatment of gestational diabetes, but it crosses the placental barrier and there is insufficient safety evidence for its use. Its pregnancy outcomes are also inferior to insulin. Metformin (C) is an oral option in the second and third trimesters but is even more likely than glyburide to cross the placental barrier. Some studies have found it reduces neonatal hypoglycemia and maternal weight gain better than insulin, but its use may be associated with premature birth. Myoinositol (D) is a supplement that has been studied to prevent and treat gestational diabetes mellitus. At this time, there is insufficient evidence to support its use.

A 57-year-old woman presents for a follow-up to a recent abnormal breast exam. She was noted to have a nontender, hard, fixed breast lump with associated enlarged axillary lymph nodes on her annual breast exam. Her mammogram was also significant for a spiculated mass highly suspicious for malignancy. What is the most common form of this patient's diagnosis? A. Inflammatory breast carcinoma B. Invasive ductal carcinoma C. Invasive lobular carcinoma D. Paget carcinoma

B. Invasive ductal carcinoma Inflammatory breast cancer (A) is a rare form of breast cancer that does not present with a breast lump, rather it presents with skin changes, such as erythema, warmth, itchiness, or peau d'orange appearance of the breast skin. Invasive lobular carcinoma (C) is the second most common form of breast cancer. Paget carcinoma (D) is a rare form of breast cancer that involves the nipple and typically presents with itchiness and rash of the nipple.

A 27-year-old G1P0 woman at 38 weeks gestation presents to her obstetrician for a routine checkup. Which of the following should the obstetrician use to assess the position of the fetus? A. Gaskin maneuver B. Leopold maneuver C. McRoberts maneuver D. Zavanelli maneuver

B. Leopold maneuver The Gaskin maneuver (A) involves placing the pregnant patient on all fours during labor in an attempt to reduce shoulder dystocia. The McRoberts maneuver (C) involves forceful flexion of the maternal legs onto the abdomen with simultaneous application of suprapubic pressure in an attempt to facilitate delivery that is complicated by shoulder dystocia. The Zavanelli maneuver (D) is a last-resort maneuver that involves replacement of the fetal head in the pelvis in preparation for cesarean delivery.

A 23-year-old woman presents to the office complaining of lower abdominal pain for the last 2 days. The pain is worse on the left, and she cannot seem to find a comfortable position. Her last menstrual period was 2 months ago, and her menses are usually every 28 days. She reports very light spotting on day 32 of her cycle. Her vital signs show a blood pressure of 121/78 mm Hg, pulse of 94 bpm, temperature of 98.6°F, and respirations of 10 breaths per minute. An abdominal exam reveals tenderness in the left lower quadrant. Bimanual exam reveals a 6 cm uterus with smooth contours. There is adnexal fullness on the left. The cervix is closed and without abnormalities. A urine human chorionic gonadotropin is positive, and serum value is 2,000 mlU/L. A transvaginal ultrasound reveals a left adnexal mass of 1.8 cm with a small amount of free fluid in the cul-de-sac. What is the most appropriate management option at this time? A. Intravenous fluids and parenteral antibiotics B. Methotrexate C. Mifepristone D. Misoprostol

B. Methotrexate Ectopic Pregnancy Risk factors include prior ectopic, PID, tubal surgery, IUD Symptoms include abdominal pain, pelvic pain, amenorrhea, or vaginal bleeding Labs will show positive pregnancy test and lower than expected serum beta-hCG levels Diagnosis is made by ultrasound Gestational sac with a yolk sac or embryo outside of the uterine cavity Free fluid with debris is suggestive to ruptured ectopic Most commonly located in a fallopian tube Treatment is methotrexate or surgery Intravenous fluids and parenteral antibiotics (A) are not indicated in this case. They may be considered in situations of a septic abortion, a medical condition where retained products of conception have caused endometritis. Since the patient described in the clinical vignette is afebrile, a septic abortion is unlikely. Mifepristone (C) is a medication commonly used in conjunction with misoprostol to induce abortion. It is not the most appropriate therapeutic option in this case. Misoprostol (D) is a prostaglandin that initiates cervical dilation and uterine contractions. It is a medication used in a variety of ways in obstetrics and gynecology. It may be used to induce abortion and for ripening the cervix prior to labor induction. It is not the most appropriate therapeutic option for a suspected ectopic pregnancy.

A 29-year-old woman who is 25 weeks pregnant presents to the office due to increased foul-smelling vaginal discharge over the past week. A physical exam reveals the presence of runny and milky discharge that has a fishy odor. The pH of the vaginal sample is 5.0, and there are clue cells identified on wet mount microscopy. Which of the following is a first-line treatment for this condition? A. Metronidazole 2 g one dose orally B. Metronidazole 250 mg orally three times daily for 7 days C. Tinidazole 1 g orally once daily for 5 days D. Tinidazole 2 g one dose orally

B. Metronidazole 250 mg orally three times daily for 7 days Metronidazole 2 g one dose orally (A) is the dosing to treat trichomoniasis, not bacterial vaginosis. Tinidazole 1 g orally once daily for 5 days (C) is appropriate for nonpregnant women only and not recommended for use during pregnancy. Tinidazole 2 g one dose orally (D) is the dosing to treat trichomoniasis, not bacterial vaginosis.

A 64-year-old woman presents with concerning breast symptoms. She reports the symptoms began nearly 2 years ago when she noticed a left-sided breast mass. Since then, she noticed her left nipple appears to be smaller than the right and she has occasional yellowish nipple discharge from the left side. On physical exam, she has a fixed, poorly defined 0.8 cm mass on the left breast. There is overlying skin erythema with an orange peel appearance, as seen in the image above. You are able to express 0.5 cc of milky-yellow fluid from the nipple, although it is somewhat retracted. She also has left-sided axillary lymphadenopathy. A mammogram is ordered. Which physical exam finding is the most concerning for inflammatory breast neoplasia? A. Nipple retraction B. Peau d'orange skin changes of the breast C. Right upper quadrant breast mass D. Serous nipple discharge

B. Peau d'orange skin changes of the breast Nipple retraction (A) may be a physical exam finding consistent with breast cancer if the tumor or mass is located behind the nipple, causing it to retract. However, nipple retraction can also be a normal physical exam finding in postmenopausal women. It can be caused by aging or breast duct ectasia. It is not one of the most concerning features of inflammatory breast cancer. A right upper quadrant breast mass (C) is a physical exam finding concerning for breast cancer in general, but it is not specific to inflammatory breast cancer. The right upper quadrant is the most common location to find breast cancer. Serous nipple discharge (D) is a concerning physical exam finding and should be evaluated appropriately. However, serous nipple discharge is not one of the most concerning features of inflammatory breast cancer. Sanguineous nipple discharge is a physical exam finding consistent with papillary breast carcinoma, a rare type of breast cancer.

What is the Chadwick sign?

Bluish hue on the cervix, which is a common sign of pregnancy.

What condition is FDA approved for treatment with anastrozole?

Breast Cancer

A 24-year-old woman presents to the emergency room with right-sided pelvic pain and vaginal bleeding. Vital signs are T of 98.4°F, HR of 108 bpm, BP of 124/78 mm Hg, RR of 20/min, and oxygen saturation of 99% on room air. The patient has right-sided adnexal tenderness on physical exam with blood in the vaginal vault. The human chorionic gonadotropin level is 6,000 mIU/mL. Her transvaginal ultrasound is shown above. Which of the following gestational ages is the most common time for this condition to present? A. 10-12 weeks B. 4-6 weeks C. 6-8 weeks D. 8-10 weeks

C. 6-8 weeks Ectopic pregnancy is defined as a pregnancy occurring outside of the uterus. Most ectopic pregnancies occur in the fallopian tubes. Risk factors include previous ectopic pregnancy (most important), prior tubal surgery, pregnancy occurring with an intrauterine device in place, and prior pelvic inflammatory disease. Lower abdominal pain and vaginal bleeding are the most classic symptoms. The vaginal bleeding is usually heavy and painful, but the quality can vary and the bleeding may be light or painless. Ectopic pregnancies occur most frequently 6 to 8 weeks after the start of the last menstrual period. Unfortunately, tubal rupture is the initial presentation in 50% of women with ectopic pregnancy. The symptoms and signs of tubal rupture may include lightheadedness, syncope, orthostasis, hypotension, and tachycardia in addition to vaginal bleeding and abdominal pain. Ectopic pregnancy is diagnosed based on clinical findings, laboratory testing, and pelvic ultrasound. It is important to correlate the human chorionic gonadotropin (hCG) levels with the transvaginal ultrasound. The discriminatory zone is the hCG level at which an intrauterine gestational sac should be visible on transvaginal ultrasound in a normal intrauterine pregnancy. The most common discriminatory zone threshold used for transvaginal ultrasounds is 2,000 mIU/mL. However, sometimes 3,500 mIU/mL is used as the threshold, which decreases the false-positive rate. Patients whose hCG level is above the discriminatory zone and do not have evidence of an intrauterine gestational sac on transvaginal ultrasound are considered to be at risk for an ectopic pregnancy. The presence of an adnexal mass in combination with the absence of an intrauterine gestational sac and hCG level above the discriminatory zone is diagnostic of an ectopic pregnancy. Patients often present with vaginal bleeding and lower abdominal pain during early pregnancy and hCG level below the discriminatory zone without signs of an intrauterine or ectopic pregnancy on ultrasound. These patients are often managed by rechecking the hCG level in 48-72 hours as long as they are hemodynamically stable without signs of an acute abdomen. The differential diagnosis in these patients includes a normal intrauterine pregnancy, abnormal intrauterine pregnancy, and ectopic pregnancy. The treatment options for an ectopic pregnancy include expectant management, methotrexate, and surgical intervention. Expectant management is not recommended for women with suspected ectopic pregnancy hCG of at least 200 mIU/mL. Methotrexate works best in patients with ectopic pregnancy who meet each of the following criteria: hemodynamically stable, normal kidney and liver function, able to comply with follow-up, pretreatment hCG < 5,000 mIU/mL, and no fetal cardiac activity on transvaginal ultrasound. However, methotrexate is often attempted in patients with higher hCG levels who are hemodynamically stable and wish to avoid surgery. Patients who are hemodynamically unstable or fail methotrexate therapy require surgical treatment.

A 21-year-old woman presents to her gynecologist extremely upset and scared. She states she was at a fraternity party last night and blacked out. When she woke up this morning, she was completely unclothed. She reports vaginal pain and bleeding. The patient is up-to-date on her hepatitis B and human papillomavirus (HPV) vaccinations. In addition to offering prophylactic treatment for HIV, which of the following antibiotics would be most appropriate for empiric coverage of sexually transmitted infections? A. Azithromycin, levofloxacin, metronidazole B. Ceftriaxone, ciprofloxacin, metronidazole C. Ceftriaxone, doxycycline, metronidazole D. Ceftriaxone, levofloxacin, metronidazole

C. Ceftriaxone, doxycycline, metronidazole Azithromycin, levofloxacin, metronidazole (A) is incorrect. Azithromycin may be used in place of doxycycline for coverage against chlamydia in the setting of sexual assault, and metronidazole is used to cover for trichomoniasis. However, levofloxacin is not recommended as first-line prophylactic coverage for sexually transmitted infections in these patients. Ceftriaxone, ciprofloxacin, metronidazole (B) and ceftriaxone, levofloxacin, metronidazole (D) are incorrect. While ceftriaxone and metronidazole are both indicated as first-line treatment to cover for gonorrhea and trichomoniasis, respectively, the fluoroquinolones (ciprofloxacin and levofloxacin) are not indicated.

A 26-year-old G4P0121 woman presents to the emergency department at 15 weeks gestation with painless vaginal spotting. She reports she has had two miscarriages during the second trimester that presented similarly. Transvaginal ultrasound confirms cervical dilation and reveals a cervical length of 20 mm. Which of the following is the recommended treatment? A. Abstinence from intercourse B. Bedrest C. Cervical cerclage D. Progesterone supplementation

C. Cervical cerclage Cervical Cerclage Procedure to stitch the cervix to prevent premature delivery or miscarriage Used for those with cervical insufficiency, short cervix, previous preterm labor Shirodkar procedure: transvaginal approach, requires paracervical dissection for placement of the suture as close to the internal cervical os as possible May be left in place if cesarean delivery is planned and future pregnancies are desired McDonald procedure: transvaginal approach, no dissection required, purse string suture placed midway into the cervical stroma as high up as possibleRemoved prior to labor Transabdominal procedure: open or laparoscopic approach, suture is placed at the cervicoisthmic junction, delivery is by cesarean May be left in place if future pregnancies are desired Abstinence from intercourse (A) and bedrest (B) are lifestyle interventions that have not been proven to reduce the risk of miscarriage or preterm birth in women with cervical insufficiency. Therefore, women with cervical insufficiency are not given any additional lifestyle interventions beyond those given to women with a healthy pregnancy. Progesterone supplementation (D) is often used to prevent preterm birth, and it is used in the setting of cervical insufficiency beginning at 16 weeks gestation.

A 24-year-old woman presents with a headache, fever, and an erythematous rash on the flanks that she noticed yesterday. She reports that she had a painless vaginal ulcer that resolved 5 weeks ago. On exam, you note a maculopapular rash on the trunk as well as on the palms and soles. Which of the following additional physical findings is most likely to be present? A. Bubo B. Chancre C. Condyloma latum D. Gumma

C. Condyloma latum Bubo (A) refers to tender unilateral or bilateral inguinal or femoral lymphadenopathy that is seen in individuals with lymphogranuloma venereum (secondary stage). Lymphogranuloma venereum is a sexually transmitted infection caused by Chlamydia trachomatis. Chancre (B) is a painless, crater-like, highly infectious lesion that is seen in individuals with primary syphilis. It appears on the genitalia 3 to 4 weeks after initial exposure. Even without treatment, the chancre resolves within 4 to 14 weeks. Gumma (D) is a soft, noncancerous growth typically seen in patients with tertiary syphilis. It is frequently found in the organs, such as the liver, brain, heart, and skin. When it is present on the skin, it usually appears as an ulcer or heaped up granulomatous lesion with a round, irregular, or serpiginous shape.

A 28-year-old nulliparous woman presents to the clinic reporting left lower quadrant pain and severe dysmenorrhea. A pelvic ultrasound reveals a left adnexal complex mass that has smooth walls with homogeneous internal echoes that have a ground-glass appearance. Which of the following ovarian masses is the most likely diagnosis? A. Corpus luteal cyst B. Dermoid cyst C. Endometrioma D. Follicular cyst

C. Endometrioma A corpus luteal cyst (A) forms when the physiologic corpus luteum fails to involute. On ultrasound, a corpus luteal cyst appears complex and heterogeneous with hypoechoic areas representing fluid and hyperechoic areas representing internal debris, such as hemorrhage. A dermoid cyst (B) is also known as a mature teratoma and arises from germ cells. Dermoid cysts consist of many materials, such as teeth and hair. For this reason, they appear heterogeneous on ultrasound with dots, fluid, and areas of acoustic shadowing. A follicular cyst (D) occurs when a physiologic follicle fails to rupture and continues to grow. A follicular cyst appears smooth and thin-walled and completely fluid-filled on ultrasound. There will be through transmission and an absence of internal echoes.

A 44-year-old woman presents to her gynecologist for her annual visit and raises concerns about recent irregularities to her menstrual cycle. She reports no sexual activity in the last year. She has not been experiencing hot flashes. She does feel moodier than usual but also reports not sleeping well recently and does not think this is related since she just changed jobs. She is otherwise well and reports no abdominal pain or abnormal bleeding. Which of the following is the best test to evaluate whether her symptoms are due to the early onset of menopause? A. Antimüllerian hormone B. Clomiphene citrate challenge test C. Follicle-stimulating hormone D. Luteinizing hormone

C. Follicle-stimulating hormone Antimüllerian hormone (A) is produced by preantral and early antral follicles. It decreases gradually as a woman's follicle pool declines with age and is undetectable at menopause, but it is primarily used to estimate ovarian reserve as part of a workup when infertility is a concern. Likewise, the clomiphene citrate challenge test (B) screens for ovarian reserve, though it involves measurement of follicle-stimulating hormone and estradiol levels, in response to oral administration of oral clomiphene citrate. Luteinizing hormone (D) is produced by the anterior pituitary gland and stimulates ovarian production of estradiol with subsequent ovulation. Measurement of luteinizing hormone is primarily used in women to predict ovulation.

A 22-year old G1P1 woman undergoes a cesarean section at 37 weeks gestation because she is HIV positive and her RNA viral load is 1,500 copies/mL. She has a recent history of herpetic breast lesions 1 month ago that have resolved. She was successfully treated for tuberculosis 2 years ago. After a 5-year history of cocaine use, she quit upon becoming pregnant, and her current urine drug screen is negative. Case management has provided the patient with resources for obtaining affordable baby formula. Which of the following in the patient's history is a contraindication for breastfeeding? A. History of cocaine use B. History of tuberculosis C. HIV status D. Recent herpetic breast lesions

C. HIV status The patient's HIV status is a contraindication to breastfeeding. Women who are HIV positive should avoid breastfeeding if replacement feeding is affordable, feasible, and sustainable. Women who are positive for human T-cell lymphotropic virus or brucellosis should not provide any breast milk to their infant. Women with active herpetic breast lesions should avoid breastfeeding until the lesions resolve but can feed their infant expressed breast milk if careful hand hygiene is observed. Women who develop varicella 5 days before delivery or 2 days after delivery or who have active tuberculosis should avoid breastfeeding but may feed their infant expressed breastmilk. Women who have H1N1 influenza should avoid breastfeeding until they are afebrile but can feed the infant expressed milk. Women who are taking illicit drugs (e.g., phencyclidine, cocaine) should avoid breastfeeding due to the risk of long-term developmental issues in their child. Infants with galactosemia should not be breastfed because they are unable to metabolize galactose and may develop failure to thrive, liver dysfunction, cataracts, and intellectual disability. History of cocaine use (A) is incorrect because the patient is not currently using cocaine. Active untreated tuberculosis, not a history of tuberculosis (B), is a contraindication to breastfeeding. The patient's recent herpetic breast lesions (D) have resolved, and she may safely breastfeed.

A 30-year-old woman is in labor and has just delivered the fetal head. The fetal head then retracts into the perineum and the rest of the fetus cannot be easily delivered with gentle and downward traction. Which of the following is the most important risk factor for shoulder dystocia? A. Diabetic pregnancies with normal fetal birth weight B. Female sex C. High birth weight in nondiabetic pregnancies D. Preterm birth

C. High birth weight in nondiabetic pregnancies Diabetic pregnancies with normal fetal birth weight (A) is incorrect. Although diabetic pregnancies are associated with an increased risk of shoulder dystocia, this is primarily driven by an increased birth weight. Female sex (B) is incorrect. Male infants are on average larger and thus associated with a higher risk of shoulder dystocia. Preterm birth (D) is incorrect. Preterm infants have a lower birth weight, and there is less risk for shoulder dystocia. Shoulder Dystocia Large fetal size Turtle sign: fetal head pulled tight against perineum Management Empty bladder McRoberts maneuver: flexing hips and legs Suprapubic pressure Delivery of posterior arm Clavicle fracture Last resort: Zavanelli maneuver (reinsert fetal head followed by C-section)

A 14-year-old girl presents to the clinic with abnormal uterine bleeding over the previous 6 months. Medical history reveals that menarche occurred 1 year ago. The patient states that she has been having menstrual bleeding at sporadic intervals greater than 28 days apart. Vital signs show BP 120/72 mm Hg, HR 77 bpm, RR 16/min, and BMI 21 kg/m2. A urine pregnancy test is negative. Which of the following is the most likely cause of this patient's abnormal uterine bleeding? A. Adenomyosis B. Hyperthyroidism C. Immature hypothalamic-pituitary axis D. Polycystic ovary disease

C. Immature hypothalamic-pituitary axis Abnormal Uterine Bleeding Sx: heavy menstrual bleeding or intermenstrual bleeding Etiology Structural causes: polyp, adenomyosis, leiomyoma, malignancy or hyperplasia (PALM) Nonstructual causes: coagulopathy, ovulatory dysfunction, endometrial, iatrogenic, not yet classified (COEIN) Laboratory assessment: pregnancy test, CBC Endometrial tissue sampling for all women age ≥ 45 years First-line imaging study: transvaginal ultrasound Hormonal treatment is combination OCPs, IV estrogen, progestins, levonorgestrel IUD Adenomyosis (A), hyperthyroidism (B), and polycystic ovary disease (D) are all causes of abnormal uterine bleeding, but all are relatively less common than an immature hypothalamic-pituitary axis in adolescent patients within the first 1-2 years of menarche.

A 60-year-old woman presents to the clinic reporting a rapidly enlarging, tender mass in her left breast. She was treated with antibiotics 3 weeks ago when she first noticed the mass, but her symptoms did not abate. She is afebrile, and other vital signs are within normal limits. On exam, an area of erythema involving the entire inferior half of the left breast is noted. The area is tender to palpation, with a firm mass underlying the erythema, and the skin of the breast is edematous, causing the pores of the skin to be very pronounced. There is a palpable, immobile, nontender left axillary node as well. Which of the following is the most likely diagnosis? A. Breast abscess B. Erythema nodosum C. nflammatory breast cancer D. Paget disease of the breast

C. Inflammatory breast cancer Inflammatory Breast Cancer Patient presents with edema and erythema that mimics mastitis Skin resembles an orange peel (peau d'orange) due to tumor emboli blocking lymphatic tissue Rare, high-grade, rapidly progressive, poor prognosis Tx: neoadjuvant chemotherapy → mastectomy with node dissection → chest wall radiation Breast abscess (A) is a common breast ailment that may present with breast pain and localized erythema. Often, in contrast to inflammatory breast cancer, the area of erythema and induration is small and well circumscribed and does not affect large swaths of the breast. Fever and malaise are frequently present. Localized lymphadenopathy may occur, but the lymph nodes are more likely to be tender and mobile. Erythema nodosum (B) is a skin manifestation of a delayed hypersensitivity reaction and most often occurs on the lower extremities, specifically the anterior shins. The lesions are erythematous, indurated, and painful. Patients will commonly report a prodrome of malaise, fever, upper respiratory symptoms, or gastrointestinal upset. The disease is self-limiting and may be secondary to certain infections or medications. Paget disease of the breast (D) primarily affects the nipple and areola, causing scaling, cracking, pruritus, bleeding, or nipple discharge. The changes begin at the nipple and then extend to the surrounding areola, often within a span of 6 to 8 months. About half of all patients with Paget disease of the breast will also have a palpable breast mass. Paget disease of the breast is a breast malignancy and may require mastectomy. Advanced disease requires radiation, as well. Inflammation and erythema of the breast tissue outside the area of the areola are not typical of Paget disease, nor is peau d'orange.

A 26-year-old G1P0 woman is in the first stage of labor. Which of the following accurately describes the fetal heart pattern shown above? A. Accelerations B. Early decelerations C. Late decelerations D. Variable decelerations

C. Late decelerations Accelerations (A) are defined by an increase of the fetal heart rate of at least 15 bpm lasting at least 15 seconds. Accelerations are reassuring. Accelerations are the measurement assessed for on a nonstress test, and if two accelerations are seen, the nonstress test is considered to be reactive. Early decelerations (B) are a normal finding during labor. Early decelerations are decelerations of the fetal heart rate that occur during labor at the same time as uterine contractions. Early decelerations occur due to compression of the fetal head. The decelerations should return to baseline as the uterine contractions end. Variable decelerations (D) occur when there are decelerations that do not correlate with the contractions on the strip. The decelerations are generally more abrupt than the gradual early decelerations seen with contractions or late decelerations seen following contractions.

A 36-year-old woman presents to her gynecologist with complaints of heavy menstrual cycles and dysmenorrhea for the past 5 months. She states her periods have previously been normal. She reports no dyspareunia or dyschezia. Pelvic examination reveals the presence of an enlarged, mobile, globular, boggy uterus with no adnexal tenderness. No cervical discharge is seen on speculum exam. Her urine pregnancy test is negative. Transvaginal ultrasound is significant for an enlarged uterus. Which of the following treatments is most appropriate for the suspected diagnosis? A. Danazol B. Estrogen-progestin contraceptive C. Levonorgestrel-releasing intrauterine device D. Mifepristone

C. Levonorgestrel-releasing intrauterine device Abnormal Uterine Bleeding Sx: heavy menstrual bleeding or intermenstrual bleeding Etiology Structural causes: polyp, adenomyosis, leiomyoma, malignancy or hyperplasia (PALM) Nonstructual causes: coagulopathy, ovulatory dysfunction, endometrial, iatrogenic, not yet classified (COEIN) Laboratory assessment: pregnancy test, CBC Endometrial tissue sampling for all women age ≥ 45 years First-line imaging study: transvaginal ultrasound Hormonal treatment is combination OCPs, IV estrogen, progestins, levonorgestrel IUD Danazol (A) is a derivative of 17-alpha-ethinyltestosterone that inhibits pituitary gonadotropin secretion, ovarian estrogen production, and endometriotic implant growth and increases free testosterone levels. It is most commonly used for the treatment of endometriosis and is known for its androgenic side effects. An estrogen-progestin contraceptive (B) is the first-line treatment for most causes of abnormal uterine bleeding. However, little data to support its efficacy in the treatment of adenomyosis. Furthermore, estrogen-progestin contraceptives may increase the size of leiomyomas. Mifepristone (D) is an antiprogestin medication previously used to reduce the size of uterine fibroids. It is not currently approved by the Food and Drug Administration for fibroid use and is associated with endometrial hyperplasia and elevated serum aminotransferases.

A 32-year-old woman presents with a complaint of increased menstrual bleeding for the past several months. She reports that her periods used to be moderate flow for 2-3 days. However, now they last for 7-10 days, and she is changing her pad or tampon every hour. Her pelvic exam is unremarkable, and no blood is seen at this time. Endometrial biopsy shows hyperplasia with atypia. A dilation and curettage reveals no evidence of endometrial carcinoma. The patient is very concerned because she has been trying to get pregnant. Which of the following is recommended for this patient? A. Anastrozole B. Hysterectomy C. Megestrol acetate D. Observation

C. Megestrol acetate Anastrozole (A) treatment has been used in patients who have hyperplasia with atypia but is not the recommended first-line treatment. Hysterectomy (B) would only be recommended for this patient if she does not wish to preserve fertility or shows no improvement after treatment with medication. It would not include an oophorectomy at this age. Observation (D) is not recommended for hyperplasia with atypia as it has a high incidence of progression to cancer. Progestin therapy is the first-line treatment in premenopausal patients wishing to preserve fertility. Abnormal Uterine Bleeding Sx: heavy menstrual bleeding or intermenstrual bleeding Etiology Structural causes: polyp, adenomyosis, leiomyoma, malignancy or hyperplasia (PALM) Nonstructual causes: coagulopathy, ovulatory dysfunction, endometrial, iatrogenic, not yet classified (COEIN) Laboratory assessment: pregnancy test, CBC Endometrial tissue sampling for all women age ≥ 45 years First-line imaging study: transvaginal ultrasound Hormonal treatment is combination OCPs, IV estrogen, progestins, levonorgestrel IUD

A 36-year-old woman is 13 weeks pregnant with twins. There are two distinct fetuses and the presence of a T sign on ultrasound. Which of the following most likely describes this patient's twin gestation? A. Dichorionic, diamniotic gestation B. Dichorionic, monoamniotic gestation C. Monochorionic, diamniotic gestation D. Monochorionic, monoamniotic gestation

C. Monochorionic, diamniotic gestation Dichorionic, diamniotic gestation (A) is incorrect because the T sign on ultrasound indicates a monochorionic, not dichorionic, pregnancy. Dichorionic, monoamniotic gestation (B) is incorrect because a gestation cannot have two distinct placentas with the fetuses sharing only one amniotic sac. Monochorionic, monoamniotic gestation (D) is incorrect because the presence of a T-sign is indicative of two separate amniotic sacs, making it a diamniotic, not monoamniotic, gestation. Multifetal gestation or multiple gestation occurs when there are multiple fetuses being carried during a pregnancy, which include twins, triplets, and higher multiples. Twins can be classified as monozygotic (originating from the division of one egg after fertilization) or dizygotic (originating from the fertilization of two different eggs). They can also be further classified by chorionicity and amnionicity. Dizygotic twins are almost always dichorionic (having two separate placentas) and diamniotic (having two separate amniotic sacs). Depending on when the fertilized egg divides, monozygotic twins can be a mixture of monochorionic (sharing one placenta) or dichorionic and monoamniotic (sharing one amniotic sac) or diamniotic. If the egg splits 0-3 days after fertilization, it will result in dichorionic, diamniotic twins. Splitting at 4-8 days after fertilization will result in monochorionic, diamniotic twins, which is the majority of monozygotic twins. Splitting at 8-12 days after fertilization, which is rare, will result in monochorionic, monoamniotic twins. Dizygotic twins result from ovulation of multiple follicles and have a familial inheritance pattern, with advanced maternal age increasing the chances of multifetal gestation due to increased levels of follicle-stimulating hormone with age.

A 27-year-old nulliparous woman presents to her gynecologist for a routine examination. She voices no concerns at this time. She is sexually active with three male partners and does not use any form of contraception. Her Pap smear report returns atypical squamous cells, cannot exclude high-grade squamous intraepithelial lesion. During colposcopy, a white, sharply demarcated lesion on the cervix is visualized after acetic acid is applied. Colposcopy is adequate, and cervical biopsy results reveal cervical intraepithelial neoplasia grade 2. The patient states she would like to have children in the future and the risks of treatment outweigh the risk of cancer. Which of the following is the best plan for management? A. Loop electrosurgical excision procedure B. Pap smear and colposcopy in 12 months C. Pap smear and colposcopy in 6 months D. Pap smear in 6 months

C. Pap smear and colposcopy in 6 months In the vignette above, an adequate colposcopy revealed cervical intraepithelial neoplasia grade 2 (CIN 2). The current recommendation states women with CIN 2 who consider the risks of treatment to outweigh the risks of cancer may be observed with a Pap smear and colposcopy in 6 months and 12 months. If the Pap smear and colposcopy are negative at both of these intervals, a Pap smear and human papillomavirus (HPV) co-testing 1 year later is recommended. If co-testing is negative, co-testing should be repeated 3 years later. If co-testing is abnormal, a colposcopy should be done. If colposcopy results reveal worsening or high-grade lesions, a repeat biopsy should be done. If CIN 2 or 3 persists for more than 2 years, treatment is recommended. If colposcopy is inadequate, if the patient has CIN 3, or if the patient has CIN 2 and opts for treatment, excision or ablation of the cervical transformation zone is indicated. Excisional treatments include loop electrosurgical excision procedure and cold knife conization. Ablative treatments include cryotherapy and laser therapy. Women with CIN 1 preceded by lesser abnormalities (e.g., atypical squamous cells of undetermined significance, low-grade squamous intraepithelial lesions, HPV 16 or 18, or persistent HPV infection) should be observed as follows: Pap smear and HPV co-testing for women ≥ 25 years of age in 12 months and Pap smear only for women 21-24 years of age in 12 months. Women ≥ 25 years of age with CIN 1 preceded by high-grade squamous intraepithelial lesion (HSIL) or atypical squamous cells, cannot exclude HSIL (ASC-H) may be observed with co-testing at 12 and 24 months, undergo a diagnostic excisional procedure, or have their previous test results reviewed to determine appropriate management. Women 21-24 years of age with CIN 1 preceded by HSIL or ASC-H may be observed with co-testing every 6 months for 2 years. Loop electrosurgical excision procedure (A) is a type of excisional treatment used for cervical dysplasia but is not appropriate for this patient at this time because she has stated the risks of the treatment outweigh the risks of cancer. If the patient had CIN 3 or persistent CIN 2, excisional treatment would be appropriate. Pap smear and colposcopy in 12 months (B) and Pap smear in 6 months (D) are incorrect because the patient should have a Pap smear and colposcopy performed in 6 months. This should be repeated again at 12 months.

A 45-year-old woman presents to the office complaining of a new vulvar lesion. She reports the lesion first appeared 2 weeks ago and has become slightly larger. It is not tender and does not bleed. Her past medical history is significant for anxiety, depression, and HIV. She has no drug allergies. Her most recent CD4+ count was 750 cells/microL. She has a history of risky sexual behavior and reports four new sexual partners in the last 2 months. She prefers not to use condoms. On physical exam, there is a 1.1 cm shallow, painless, erythematous ulceration on the mucosal surface of her left labia. Serology testing, including a CD4+ count, is ordered, and results are pending. In the meantime, what is the most appropriate pharmacotherapeutic option for this patient? A. Doxycycline 100 mg orally for 14 days B. Imiquimod 1 packet tid for 16 weeks C. Penicillin 1 dose of 2.4 million units D. Valacyclovir 1 g bid for 10 days

C. Penicillin 1 dose of 2.4 million units Primary Syphilis Sx: painless ulcer that forms 3 weeks post sexual activity and disappears in 3-6 weeks PE: vaginal, anal, or oral chancre - painless, punched-out lesion with a raised margin, lymphadenopathy Dx: darkfield microscopy, RPR/VDRL confirmed by FTA-ABS Caused by spirochete Treponema pallidum Tx: single IM injection of benzathine penicillin Mnemonic: syphilis is painless ulcer Doxycycline 100 mg orally for 14 days (A) is an available treatment option for primary syphilis but should only be used if the patient has a penicillin allergy. It is not the best therapeutic option in this case. Imiquimod 1 packet tid for 16 weeks (B) is an appropriate pharmacotherapeutic option for genital warts caused by human papillomavirus. In the clinical vignette, the patient's physical exam findings are most consistent with a chancre, as seen in primary syphilis. Genital warts present as 1-2 mm verrucous, nontender, flesh-colored or pink papules that are nontender. Imiquimod is not an appropriate treatment option for a chancre. Valacyclovir 1 g bid for 10 days (D) is an appropriate pharmacotherapeutic option for genital herpes simplex virus. While genital herpes often presents as shallow, erythematous ulcers, they are typically exquisitely painful and appear in clusters (unlike chancres, which are single, painless lesions). While there are options for serologic testing, viral culture of the suspicious lesion is the diagnostic modality of choice.

A 24-year-old woman at 37 weeks gestation presents to the office for a routine obstetrics exam. An ultrasound reveals an amniotic fluid index of 4 cm and no signs of fetal abnormalities. Which of the following is a risk factor for this condition? A. Fetal esophageal atresia B. Fetal hyperglycemia C. Preeclampsia D. Trisomy 21

C. Preeclampsia Oligohydramnios is characterized by a lower than normal volume of amniotic fluid, which can lead to underdevelopment of fetal lung tissue as well as fetal death. Amniotic fluid is necessary for the proper growth and development of the fetus by providing a protective cushion for the fetus, allowing fetal length maturity, and providing a barrier against infection. The normal volume of fluid varies throughout the pregnancy and peaks at around 800-1,000 mL near term. The most common cause of oligohydramnios is the rupture of membranes, but this is not the only cause. When the fetus is near term, it swallows amniotic fluid and produces urine. Therefore, a low amniotic fluid level can be caused by the absence of fetal urine production or fetal urinary tract blockage. Any anomaly that affects the fetal urinary tract, such as renal agenesis, polycystic kidneys, or obstructive lesions, can lead to this condition. It can also be caused by placental insufficiency due to conditions such as preeclampsia, chronic hypertension, and other maternal vascular disease or when the pregnancy is extended too long (i.e., beyond 42 weeks gestation). The mortality rate of oligohydramnios is high, and it is associated with a poor prognosis since this condition can lead to compression of the fetal abdomen, pulmonary underdevelopment, fetal heart conduction abnormalities, umbilical cord compression, and intrauterine growth restriction. Oligohydramnios can be evaluated by ultrasonography, which will show an amniotic fluid index < 5 cm or a single deepest pocket that is < 2 cm in depth. Women with diagnosed oligohydramnios should be put on bed rest with adequate hydration to promote the production of amniotic fluid. A thorough evaluation of the fetal anatomy should be done, as well as genetic testing as indicated. Fetal esophageal atresia (A), fetal hyperglycemia (B), and trisomy 21 (D) are risk factors associated with polyhydramnios (an overabundance of amniotic fluid), not oligohydramnios.

A 20-year-old woman presents to her primary care provider in January with recurrent episodes of increased irritability, anxiety, and sadness for the past 3 months. She states the holidays are a stressful time for her because she does not get along well with her family. Upon further questioning, she states her mood seems normal most of the time, then all of a sudden, her symptoms will begin and last for almost a week. Which of the following is the most likely diagnosis? A. General anxiety disorder B. Premenstrual dysphoric disorder C. Premenstrual syndrome D. Seasonal affective disorder

C. Premenstrual syndrome Premenstrual Syndrome 1-2 weeks prior to menstrual cycle Sleep disturbances, decreased focus, emotional lability, breast tenderness, or HA that resolves after menses begins Treatment is decreased caffeine intake, exercise, stress reduction, NSAIDs, SSRIs, OCPs Symptoms do not hinder personal or professional life (unlike premenstrual dysphoric disorder) General anxiety disorder (A) is characterized by persistent and excessive anxiety and worry occurring more days than not for ≥ 6 months that interferes with social, occupational, or other important areas of life. Patients must also present with three of the following six symptoms: restlessness, fatigue, difficulty concentrating, irritability, muscle tension, or sleep disturbance. Premenstrual dysphoric disorder (B) is a more severe form of premenstrual syndrome and is characterized by the presence of physical and behavioral symptoms for most of the preceding year. For a diagnosis of premenstrual dysphoric disorder, the patient must have five or more symptoms present in the luteal phase that resolves within a few days after the onset of menses. The patient in the vignette has only been having symptoms for 3 months and only has three of the criteria for premenstrual dysphoric disorder (irritability, anxiety, sadness). Seasonal affective disorder (D) is a subset of major depressive disorder and bipolar disorder. It is characterized by recurrent episodes and remission of major depression, mania, or hypomania with seasonal onset.

A 25-year-old G2P1 woman at 20 weeks gestation presents for a routine visit. Her blood pressure at this visit is 152/92 mm Hg. A repeat blood pressure taken the following day is 148/92 mm Hg. Which of the following additional findings is required for a diagnosis of preeclampsia? A. A third single systolic blood pressure reading ≥ 140 mm Hg B. Protein to creatinine ratio < 0.2 C. Proteinuria ≥ 300 mg in a 24-hour urine collection D. Thrombocytosis > 500,000/mcL

C. Proteinuria ≥ 300 mg in a 24-hour urine collection Preeclampsia Pregnancy > 20 weeks gestation or postpartum Visual disturbances, severe headaches, or asymptomatic Evaluation will show new-onset hypertension (≥ 140/90 mm Hg) with either proteinuria (≥ 300 mg/24 hr or urine protein: creatinine ratio ≥ 0.3) OR significant end-organ dysfunction Treatment: delivery at 37 weeks (without severe features) and 34 weeks (with severe features) AND prevention of seizures with magnesium sulfate and prevention of permanent maternal organ damage New-onset hypertension < 20 weeks gestation: suspect molar pregnancy A third systolic blood pressure ≥ 140 mm Hg (A) is not required to meet the criteria for hypertension in preeclampsia. Two systolic blood pressures ≥ 140 mm Hg taken at least 4 hours apart after 20 weeks of gestation in addition to either proteinuria or end organ dysfunction or both is required to make the diagnosis. Protein to creatinine ratio < 0.2 (B) is normal, while a ratio of > 0.3 indicates excess proteinuria. Thrombocytosis > 500,000/mcL (D) is not a criteria for preeclampsia. Thrombocytopenia may be used to correlate the severity of preeclampsia, and platelet levels < 100,000/mcL are an indicator of severe preeclampsia.

A 25-year-old woman presents to the clinic for a routine wellness examination. She reports her husband is routinely controlling and humiliating her. Her husband has isolated her from her friends and family and manipulates her by withholding access to money. Which of the following is the most likely diagnosis? A. Physical intimate partner violence B. Post-traumatic stress disorder C. Psychological intimate partner violence D. Sexual intimate partner violence

C. Psychological intimate partner violence Physical intimate partner violence (A) is characterized by intentionally using physical force to cause harm, injury, disability, or death. Examples may include scratching, pushing, biting, strangling, punching, burning, using a weapon, and using restraints. Post-traumatic stress disorder (B) manifests with cognitive, affective, and behavioral responses to stimuli that remind someone of previously experienced trauma. The diagnosis also requires symptoms continue for at least 1 month following the trauma and impair social or occupational function. Post-traumatic stress disorder may occur following intimate partner violence. Sexual intimate partner violence (D) can manifest in multiple ways. These include the use of physical force to get a person to engage in a sexual act against their will, an attempted or completed sexual act in which a person is unable to give consent, or abusive sexual contact.

A 26-year-old woman presents to her primary care provider with increasing symptoms of anger, irritability, difficulty concentrating, feeling overwhelmed, and insomnia that have been causing significant interference to her work and daily activities. She has been recording the timing of her symptoms for the past 12 months and noticed her symptoms begin the week prior to her menses and resolve 2 to 3 days after menses. Her thyroid levels are within normal limits. Dysthymic disorder and major depressive disorder are ruled out. Which of the following medications is recommended as the first-line treatment for this patient? A. Alprazolam B. Leuprolide C. Sertraline D. Vitamin B6

C. Sertraline Alprazolam (A) has previously been used as adjunct therapy for treatment of premenstrual dysphoric disorder but is no longer recommended because the potential adverse effects including risk of addiction and misuse outweigh the benefits of treatment. Leuprolide (B) is a GnRH agonist that may be used in the treatment of premenstrual dysphoric disorder if the patient is unresponsive to several SSRIs or oral contraceptives. GnRH agonists should be given with estrogen-progestin add-back therapy to prevent menopausal symptoms, osteopenia, or osteoporosis. Vitamin B6 (D), also known as pyridoxine, is not recommended for treatment of premenstrual dysphoric disorder. High-dose pyridoxine has been shown to cause peripheral neuropathy.

A 72-year-old woman presents to the office with complaints of a new vaginal mass. While she was cleaning her house earlier in the day, she reports feeling a sudden fullness vaginally. She is worried her bladder is falling out. She is experiencing some discomfort that is worse when she is sitting. On physical exam, the external genitalia appear normal, and there is no frank blood. When the patient performs a Valsalva maneuver, the uterine cervix protrudes through the vaginal introitus by 5 mm. When relaxed, the prolapse is completely reduced. Bimanual exam reveals a 4 cm uterus with a smooth contour and no adnexal masses. There is anterior vaginal wall laxity. Which stage of pelvic organ prolapse is this patient experiencing? A. Stage 1 B. Stage 2 C. Stage 3 D. Stage 4

C. Stage 3 A stage 1 (A) prolapse is characterized by a herniation that is further than 1 cm from the vaginal introitus. Oftentimes, stage 1 prolapse is not felt by the patient and is only found on vaginal or bimanual exams. A stage 2 (B) prolapse is 1 cm or closer to the vaginal introitus but does not protrude through the introitus with any amount of Valsalva. At this stage, the prolapse may be symptomatic. Patients may feel vaginal fullness, pressure, pelvic pain, or a sensation of sitting on a ball. A stage 4 (D) prolapse would indicate tissue completely outside of the vaginal introitus with or without Valsalva. Stage 4 prolapse can cause significant discomfort and trauma to the vaginal wall and exposed tissue. A pessary may be an available option for stage 4 prolapse, but finding an appropriate size may be difficult.

A 30-year-old woman presents to the clinic complaining of infertility. She says she has had painful menses and pain with intercourse for years. Transvaginal ultrasound shows a left-sided ovarian cystic mass with homogeneous low-medium-level echoes. Which of the following exam findings supports the most likely diagnosis? A. Bleeding from the cervical os B. Endocervical purulent discharge C. Tenderness in the posterior vaginal fornix D.. Vaginal dryness and loss of rugae

C. Tenderness in the posterior vaginal fornix Endometriosis Patient presents with pre- or mid-cycle dysmenorrhea, dyspareunia, dyschezia (painful bowel movement) PE may show uterosacral nodularity or a fixed or retroverted uterus or adnexal mass Definitive diagnosis is made by laparoscopy Most common site is ovaries Tx: NSAIDs, COCs, depot medroxyprogesterone acetate, GnRH agonists, surgery Bleeding from the cervical os (A) is not an exam finding associated with endometriosis. Endocervical purulent discharge (B) can be seen with infectious processes such as cervicitis or pelvic inflammatory disease. Vaginal dryness and loss of rugae (D) are suggestive of an estrogen deficiency, such as in premature ovarian failure or menopause.

A 37-year-old woman presents to the office to ask about female contraceptive barrier methods and wants to be fitted for a cervical cap or diaphragm. Which of the following is a side effect of using these devices? A. Menorrhagia B. Pelvic inflammatory disease C. Urinary tract infection D. Urogenital fistula formation

C. Urinary tract infection Both require professional fitting since improper sizing can cause erosions to the vagina or cervix. The advantages are that they are discreet, controlled by the woman, and are not typically felt by either partner during intercourse, they do not have any systemic side effects, and they protect against pelvic infections and cervical dysplasia when used correctly. A diaphragm is a shallow silicone cup that should be inserted less than 1 hours prior to intercourse up to 6 hours before intercourse and must be kept in place for at least 6 hours after intercourse. If it is placed more than 1 hour before intercourse, another applicator full of spermicide or vaginal pH regulator gel needs to be inserted into the vagina for maximum effect. A cervical cap is a latex cup that fits over the base of the cervix. An advantage of the cervical cap over the diaphragm is that it can be inserted for up to 6 hours prior to intercourse and does not require reapplication of spermicide if time has elapsed between placement and intercourse. Cervical caps have a higher failure rate than diaphragms. Additionally, parous women have higher failure rates than nulliparous due to the change in size of the cervix after childbirth, causing a poor fit. Applying spermicide will help improve outcomes for both devices. Effectiveness of both devices depends on the experience and motivation of the user, care of the device, and use of spermicide. Side effects of both include increased risk of urinary tract infections (especially with diaphragm use during multiple acts of intercourse) and a small risk of toxic shock syndrome. Therefore, a diaphragm or cervical cap should not be put in for more than 12-18 hours. They also should not be used during menstruation. Menorrhagia (A) is a side effect of copper intrauterine device but not diaphragms or cervical caps. Pelvic inflammatory disease (B) can occur hypothetically because these methods do not fully protect against sexually transmitted infections such as Chlamydia trachomatis, which can cause pelvic inflammatory disease, but this condition is not a direct side effect of using these devices. Urogenital fistula formation (D) is not a side effect of diaphragm or cervical cap use but is typically a result of iatrogenic causes (e.g., cesarean section).

A 22-year-old woman presents to her primary care provider with recurrent bilateral breast pain each month. Physical exam reveals no masses, skin changes, nipple discharge, or axillary lymphadenopathy. Which part of her menstrual cycle is the most likely timing for this pain to occur in? A. During menses B. First week after menses C. Week before menses D. Week of ovulation

C. Week before menses Fibrocystic Breast Changes Risk factors: women 30−50 years old Sx: intermittent breast pain and tenderness that peak before each menstruation Ultrasound may show dense, prominent, fibroglandular tissue with cysts but no discernible mass Most commonly caused by fluctuating hormone levels during menstrual cycles Treatment is well-fitting supportive bras, applying heat to the breasts, or over-the-counter pain relievers Most common lesion of the breastFibrocystic changes are generally benign and do not increase risk for breast cancer During menses (A), the first week after menses (B), and the week of ovulation (D) are each incorrect. Cyclic breast pain classically occurs the week before menses and improves with the start of menses.

What type of twins occurs with division of a fertilized egg 13 days or later after fertilization?

Conjoined twins

A 28-year-old woman who is morbidly obese presents to her primary care office with reports of irregular menstrual periods. On physical exam, there is increased hair growth to her upper lip, jawline, and abdomen. Her most recent labs are consistent with an elevated hemoglobin A1C, elevated LH, and elevated testosterone. Which of the following would most likely be found on this patient's pelvic ultrasound? A. Focal heterogeneous mass B. Free fluid within the posterior cul-de-sac C. Thickened endometrial stripe D. "String of pearls" sign

D. "String of pearls" sign Polycystic Ovary Syndrome (PCOS) Amenorrhea, obesity or overweight, hirsutism PE will show bilateral ovarian enlargement, acanthosis nigricans Labs will show high LH to FSH, androgen excess Most commonly caused by insulin resistance Treatment is combination oral contraceptive pills, lifestyle changes, metformin Most common cause of infertility Focal heterogeneous mass (A) would be noted on a pelvic ultrasound if the patient were to have uterine fibroids, also known as a leiomyoma. Free fluid within the posterior cul-de-sac (B) is a nonspecific finding that may suggest findings such as pelvic inflammatory disease, a ruptured ovarian cyst, or ectopic pregnancy. Thickened endometrial stripe (C) is consistent with the diagnosis of endometrial hyperplasia or cancer.

A 36-year-old G1P0 woman presents to the clinic for a prenatal evaluation. Recently, she has had severe nausea and vomiting. Physical exam reveals a larger than expected uterine size for gestational age, with bilateral adnexal masses noted on bimanual exam. Ultrasound imaging of the uterus is shown above. When is the earliest this condition can be diagnosed? A. 12 weeks gestation B. 20 weeks gestation C. 28 weeks gestation D. 8 weeks gestation

D. 8 weeks gestation Gestational trophoblastic disease is diagnosed as early as 8 weeks gestation, thus it can be diagnosed before 12 (A), 20 (B), and 28 weeks gestation (C). Complete molar pregnancies may become more symptomatic during the second trimester and diagnostic imaging may be delayed secondary to this fact, but prompt evacuation of uterine contents is best completed as early as possible, with fewer complications arising after first-trimester treatment.

A 28-year-old otherwise healthy woman presents to the gynecology clinic with 24 hours of right-sided breast pain, redness, and swelling. She is 5 weeks postpartum. Vital signs are T of 100.6°F, HR of 98 bpm, BP of 120/80 mm Hg, RR of 20/min, and oxygen saturation of 98% on room air. Breast examination reveals a local area of erythema and firm edema on the right breast. Which of the following is the recommended treatment? A. Cessation of breastfeeding and clindamycin B. Cessation of breastfeeding and dicloxacillin C. Continuation of breastfeeding and clindamycin D. Continuation of breastfeeding and dicloxacillin

D. Continuation of breastfeeding and dicloxacillin Mastitis Patient will be a breastfeeding mother Breast erythema, tenderness, fever Most commonly caused by Staph. aureus Management includes cool compresses and analgesics between feedings Antibiotics: dicloxacillin, cephalexin, TMP-SMX (MRSA), clindamycin (PCN allergy) Continue breast feeding to avoid progression to abscess Cessation of breastfeeding and clindamycin (A) is incorrect because woman with infective lactational mastitis are supposed to continue breastfeeding to allow drainage of the milk and because the patient does not have an indication for coverage for methicillin-resistant Staphylococcus aureus. Cessation of breastfeeding and dicloxacillin (B) is incorrect because women with infective lactational mastitis are supposed to continue breastfeeding to allow drainage of the milk. Continuation of breastfeeding and clindamycin (C) is incorrect because the patient does not have an indication for coverage for methicillin-resistant Staphylococcus aureus.

A 35-year-old G2P1 woman at 41 weeks gestation is in a prolonged second stage of labor. Delivery of the fetus is blocked by perineal tissue. A category III tracing is unresponsive to resuscitative measures, and the obstetrician decides to perform an episiotomy. Which of the following is the preferred type of episiotomy to minimize the risk of anal sphincter laceration? A. Anterior episiotomy B. J incision C. Lateral episiotomy D. Mediolateral episiotomy

D. Mediolateral episiotomy An anterior episiotomy (A), also known as deinfibulation, is only used in patients with a history of female circumcision or female genital mutilation. The incision is made in the midline through the fused labia majora toward the pubis to reveal the external urethral meatus. A J incision (B) is less commonly used than a median or mediolateral incision and is performed by starting at the fourchette, extending caudally in the midline, and then curving laterally at an angle. A lateral episiotomy (C) is rarely used. It is performed by beginning the incision 2 cm lateral to midline and extending toward the ischial tuberosity.

A 70-year-old woman presents to her gynecologist with concerns about a pruritic vaginal lesion. She was prescribed a vaginal cream for eczema by her primary care provider 6 weeks ago, but her symptoms have not improved. Physical examination is significant for a well-demarcated, slightly raised, erythematous vulvar lesion with an eczematoid appearance, as shown above. The rest of her physical examination is unremarkable. Which of the following is the most likely diagnosis? A. Atrophic vaginitis B. Bartholin gland cyst C. Lichen planus D. Paget disease of the vulva

D. Paget disease of the vulva Atrophic vaginitis (A), also known as vulvovaginal atrophy, is characterized by vaginal dryness, burning, and dyspareunia. It is most commonly seen in menopausal women and is the result of decreased estrogen levels. A Bartholin gland cyst (B) occurs when the Bartholin duct becomes obstructed, resulting in local or diffuse vulvar edema. Complications include the development of a Bartholin abscess. Treatment includes incision and drainage. The placement of a Word catheter is indicated for masses ≥ 3 cm. Lichen planus (C) of the vulva is characterized by an erosive, papular, or hypertrophic lesion on the vulva and may result in severe tissue destruction, pain, and urinary or sexual dysfunction. Patients will typically present with manifestations of lichen planus on other cutaneous and mucosal surfaces, such as the skin, oral cavity, scalp, and nails. The five Ps (pruritic, planar, purple, polygonal, and papular) can be used to remember the clinical presentation of lichen planus. Treatment for lichen planus of the vulva includes avoiding offending medications and applying topical corticosteroids.

A 67-year-old G6P5 woman presents to the clinic complaining of pelvic pressure that has progressively worsened for months. She also complains she cannot empty her bladder completely. She is currently sexually active. Physical examination reveals collapse of the anterior vaginal wall. Which of the following is the recommended initial treatment? A. Anterior colporrhaphy B. Heavy resistance-based exercise C. Obliterative surgery D. Pessary

D. Pessary Anterior colporrhaphy (A) is a surgical procedure often used to fix a cystocele by repairing defects in the anterior or posterior vaginal wall. However, initial conservative treatment of pelvic organ prolapse is recommended before surgical treatment. Heavy resistance-based exercise (B) is incorrect because it could increase intra-abdominal pressure and worsen pelvic organ prolapse. However, aerobic exercise and dietary changes to promote weight loss are helpful. Obliterative surgery (C) is a surgical approach to pelvic organ prolapse that is useful in women who have no desire for future sexual intercourse, whereas the patient in the vignette is sexually active. The benefits of obliterative surgery include lower rates of prolapse recurrence and less perioperative morbidity.

A 33-year-old G2P1 woman presents to the emergency department reporting severe abdominal pain and dark red vaginal bleeding. She is currently 37 weeks pregnant and appears to be in labor. On exam, the baby is bradycardic, and a bedside ultrasound shows separation of the placenta edge. What is the greatest risk factor for this patient's condition? A. Abdominal trauma B. Cocaine use C. Eclampsia D. Previous placental abruption

D. Previous placental abruption Although abdominal trauma (A), cocaine use (B), and eclampsia (C) are known risk factors for placental abruption, they are neither the greatest nor the most common risk factors. Placental Abruption Patient will be in third trimester History of hypertension, trauma, or cocaine use Painful vaginal bleeding Labs will show hypofibrinogenemia Tx: fetal monitoring, hemodynamic stabilization, delivery

Which hormone is responsible for uterine ripening to allow proper implantation of a fertilized ovum? A. Estrogen B. Follicle-stimulating hormone C. Luteinizing hormone D. Progesterone

D. Progesterone Estrogen (A) is the main hormone that works during the follicular phase to stimulate the proliferation of the endometrial lining but does not cause ripening of the uterine lining for implantation. Follicle-stimulating hormone (B) stimulates granulosa cells in the ovaries to produce estrogen but does not play a direct role in the preparation of the uterine lining for implantation. Luteinizing hormone (C) stimulates ovarian theca cells to produce progesterone but does not play a direct role in the preparation of the uterine lining for implantation.

A 30-year-old nulliparous woman presents to her gynecologist with prolonged and heavy menstrual bleeding for the past 6 weeks. Upon further questioning, she states she has also had abdominal discomfort. She is not currently taking any medications. She is sexually active with three male partners and does not use contraception. Pelvic examination is significant for a mildly enlarged uterus (7 cm) without cervical motion tenderness or adnexal masses. Transvaginal ultrasound confirms the suspected diagnosis. Which of the following treatments is most appropriate for the patient at this time? A. Azithromycin only B. Ceftriaxone and azithromycin C. Gonadotropin-releasing hormone agonist D. Progestin-releasing intrauterine device

D. Progestin-releasing intrauterine device Ceftriaxone and azithromycin (B) is preferred over azithromycin only (A) for the treatment of vaginitis secondary to Chlamydia trachomatis or Neisseria gonorrhoeae due to an increased risk of co-infection. However, these antibiotics have no role in the treatment of leiomyomas. A gonadotropin-releasing hormone agonist (C) is primarily used for preoperative or transitional therapy and reduces the size of leiomyomas. However, it is more expensive than first-line agents and is associated with hypoestrogenic effects such as bone loss. Uterine Fibroids (Leiomyoma) Common during reproductive-ages Menorrhagia and dysmenorrhea PE will show a enlarged, asymmetric, and nontender uterus Diagnosis is made by pelvic ultrasound Majority do not require surgical or medical treatment Severe cases: myomectomy (fertility can be preserved) or hysterectomy

Which of the following hormones is responsible for breast milk production following delivery? A. Estrogen B. Oxytocin C. Progesterone D. Prolactin

D. Prolactin Estrogen (A) and progesterone (C) are responsible during gestation for increasing ductal proliferation and alveolar gland maturation. These hormones inhibit prolactin's effect on mammary tissues and decrease following delivery, thereby allowing increased milk production. Oxytocin (B) stimulates the letdown response that allows for ejection of breast milk, but it does not directly contribute to the production of breast milk.

A 55-year-old woman presents to the office due to vaginal fullness and pressure that has gotten worse over the past year. She feels a bulge at the opening of her vagina especially after being on her feet for several hours. She is also having more difficulty urinating. She does not have any problems with defecation and does not report vaginal bleeding or pain. She is widowed and has six adult children who were delivered vaginally. Which of the following is the most appropriate next step in treatment? A. Prescribe her a topical estrogen cream B. Prescribe her nitrofurantoin C. Refer her to a gynecologic surgeon for a surgical consult D. Refer her to a specialist for a pessary fitting

D. Refer her to a specialist for a pessary fitting Prescribe her a topical estrogen cream (A) is incorrect, since there is no evidence that this is helpful for the treatment of uterine prolapse. Prescribe her nitrofurantoin (B) is incorrect because nitrofurantoin is an antibiotic used for the treatment of cystitis, which is not consistent with the patient's clinical scenario. Refer her to a gynecologic surgeon for a surgical consult (C) is incorrect because surgery is only indicated if she has failed or declined conservative management first.

A 30-year-old woman presents to her gynecologist for a colposcopy after an abnormal Pap smear. Examination of the cervix reveals dense, well-defined acetowhite areas near the squamocolumnar junction after acetic acid is applied. A biopsy of the dysplastic lesion is obtained. What is the most common type of carcinoma given the patient's most likely diagnosis? A. Adenocarcinoma B. Epithelial carcinoma C. Small cell carcinoma D. Squamous cell carcinoma

D. Squamous cell carcinoma Adenocarcinoma (A) is the second most common type of cervical carcinoma after squamous cell carcinoma. Adenocarcinoma is also the most common type of endometrial cancer. Epithelial carcinoma (B) is the most common type of ovarian cancer. Small cell carcinoma (C) of the cervix is rarely seen in cervical malignancies and accounts for approximately 2% of cervical carcinomas.

A 25-year-old G1P0 woman at 9 weeks gestation by last menstrual period presents to her obstetrician with nausea and vomiting. The patient states that she has vomited at least 5 times each day for the past 4 weeks and has lost about 9 pounds over that time period. Which of the following is most likely to be associated with the patient's condition? A. Anion gap metabolic acidosis B. Hyperchloremic alkalosis C. Hyperkalemia D. Starvation ketosis

D. Starvation ketosis Hyperemesis Gravidarum Peak incidence: weeks 8-12 Weight loss Hypokalemia Ketonemia Rx: IVF with 5% dextrose, antiemetics Anion gap metabolic acidosis (A) and hyperchloremic alkalosis (B) are not associated with hyperemesis gravidarum. Hypochloremic metabolic alkalosis is the most common acid-base abnormality seen in patients with hyperemesis gravidarum. Hypokalemia secondary to vomiting, not hyperkalemia (C), is consistent with hyperemesis gravidarum.

A 29-year-old G1P0 woman at 11 weeks gestation presents to the ED with vaginal pain and bleeding. Transvaginal ultrasound reveals the presence of an intrauterine pregnancy. Fetal heart rate is 138 bpm. Her serum chorionic gonadotropin level was 39,488 mIU/mL 5 days ago and is currently 153,867 mIU/mL. Pelvic examination reveals the presence of a closed cervical os with a small amount of blood in the vaginal canal. What is the most likely diagnosis? A. Incomplete abortion B. Inevitable abortion C. Missed abortion D. Threatened abortion

D. Threatened abortion Spontaneous abortion, also known as a miscarriage, is a pregnancy loss occurring before 20 weeks gestation. The different types of spontaneous abortion are threatened, inevitable, incomplete, missed, and complete abortion. Risk factors include maternal disease (e.g., diabetes mellitus, thyroid disease, thrombophilia, lupus anticoagulant), being severely overweight or underweight, structural abnormalities of the uterus, and exposure to teratogens or infections. Diagnosis is made by pelvic examination, serially decreasing human chorionic gonadotropin levels, and transvaginal ultrasound showing inappropriate development or fetal demise. A threatened abortion is characterized by vaginal bleeding with or without uterine contractions in the presence of a closed cervical os. These patients should be followed closely with a weekly ultrasound until a viable pregnancy is confirmed or ruled out. An inevitable abortion is characterized by vaginal bleeding and a dilated cervical os without passage of any fetal tissue. Products of conception can be felt or visualized through the cervical os. An incomplete abortion is characterized by vaginal bleeding and a dilated cervical os with the passage of some but not all of the products of conception. A missed abortion is characterized by the death of the fetus before 20 weeks gestation, retained products of conception, and a closed cervical os. Patients with a missed abortion may or may not have symptoms. An inevitable, incomplete, or missed abortion is managed with surgical uterine evacuation (e.g., dilation and curettage), pharmacologic uterine evacuation (e.g., mifepristone followed by misoprostol or misoprostol only), or expectant management. Retained products of conception after administration of misoprostol may be treated with surgical evacuation or a second round of misoprostol. Expectant management is an option for patients at < 14 weeks gestation who are stable and have no signs of infection. Retained products of conception after 4 weeks of expectant management should be treated with surgical evacuation. A complete abortion is characterized by complete passage of the products of conception out of the uterus and cervix. On physical examination, the cervical os is closed and the uterus is small. Rh-negative patients who have had a spontaneous abortion should receive Rh immune globulin to prevent alloimmunization. A 50 mcg dose is given if the abortion occurred before 13 weeks gestation. Otherwise, the standard 300 mcg dose is given. Serum human chorionic gonadotropin typically returns to normal within 6 weeks after a completed abortion and may be monitored serially after expectant management or pharmacologic evacuation. Pelvic rest is recommended for 2 weeks following an abortion. Counseling should be offered to patients to address grief and provide emotional and psychological support. An incomplete abortion (A) or inevitable abortion (B) would present with a dilated cervical os. A missed abortion (C) would present with a closed cervical os, but no fetal heart rate would be detected. Additionally, the serum chorionic gonadotropin level would be decreasing rather than increasing. In a viable pregnancy, the serum chorionic gonadotropin level is expected to double every 48 hours.

A 26-year-old woman presents to the office due to worsening depression, mood swings, anxiety, low energy, and poor concentration that begins about 2 weeks before her period and lasts until her period starts. She states she has had these symptoms since she was a teenager, but lately they have gotten so severe she has had to take many days off work and is currently at risk of losing her job. A physical examination is unremarkable. Which of the following diagnostic tests should be ordered in the workup of her condition? A. Adrenocorticotropic hormone stimulation test B. Fasting plasma glucose test C. Overnight sleep study D. Thyroid-stimulating hormone test

D. Thyroid-stimulating hormone test To diagnose PMDD, other underlying conditions that exhibit similar symptoms must be evaluated for, including thyroid disease, anemia, perimenopause, and menopause. Therefore, a thyroid-stimulating hormone test, human chorionic gonadotropin, complete blood count, and follicle-stimulating hormone level should be ordered. Treatment of this condition has both nonpharmacologic and pharmacologic components. Nonpharmacologic therapy includes acupuncture, relaxation techniques, light therapy, and cognitive behavior therapy. Aerobic exercise can also be helpful, especially in patients who have depression or bloating as predominant symptoms. Pharmacologic therapy includes the use of hormones (e.g., combined oral contraceptives), diuretics (e.g., spironolactone), nonsteroidal anti-inflammatory drugs (e.g., naproxen), and antidepressants (e.g., selective serotonin reuptake inhibitors). Hysterectomy plus bilateral oophorectomy have shown to be curative for patients with PMDD. Adrenocorticotropic hormone stimulation test (A) is used to diagnose Addison disease. Although it is associated with fatigue and myalgia (features of PMDD), Addison disease is also characterized by skin hyperpigmentation and progressive weakness and is not an underlying disease commonly considered as a differential diagnosis for PMDD. A fasting plasma glucose test (B) can be used to evaluate for diabetes mellitus, a diagnosis that can cause similar symptoms as PMDD, such as appetite changes, easy fatigability, and decreased energy, but is unlikely since the patient's symptoms are cyclical and related to her menstrual cycles. Overnight sleep study (C) is helpful to rule out obstructive sleep apnea, a major cause of fatigue, depression, and irritability, but would not explain the cyclical nature of the patient's symptoms and their association with her menstrual cycle.

A 25-year-old woman presents to the emergency department with a painful, swollen left breast and low-grade fever. She reports she developed mastitis while breastfeeding about 2 weeks ago that initially improved with antibiotics. She reports smoking one pack of cigarettes daily but always outside, away from the baby. She is taking no medications other than acetaminophen. She no longer has pain with breastfeeding but has developed a hot, painful, swollen area on her breast. On exam, she is obese but otherwise appears well. Her left breast has a localized, indurated area of about 3 cm diameter to the left of the areola, which is warm, tender to palpation, and fluctuant. Bedside ultrasound confirms the diagnosis and also facilitates aspiration of fluid. Which of the following elements of this patient's history puts her at greatest risk for this being a recurrent problem? A. Age B. Breastfeeding C. Obesity D. Tobacco use

D. Tobacco use Breast Abscess Patient presents with fever, malaise, painful breast lump PE will show fluctuant, tender, palpable mass Most commonly caused by Staph aureus Usually a complication of mastitis Age (A) is a risk factor only for women developing lactational mastitis, with maternal age > 30 years old increasing risk. It is not associated with abscess recurrence. Breastfeeding (B) is not a risk factor for breast abscess recurrence, though women with lactational mastitis are at increased risk for abscess. Lactational abscesses are less likely to be recurrent. Obesity (C) is a risk factor for primary breast abscess in nonlactating women but is not associated with recurrence.

A 24-year-old woman presents to the emergency department with intense right lower quadrant pain for the past 12 hours. The patient is afebrile, tachycardic, and normotensive. Her last menstrual period was 8 weeks ago. She reports no vomiting or diarrhea but reports nausea and dyspareunia. Physical exam reveals right lower quadrant tenderness with guarding and without rebound tenderness. Vaginal exam reveals a closed cervical os and a right adnexal fullness on bimanual palpation. Which of the following represents the best next step in the diagnosis of this patient's pathology? A. Abdominal X-ray B. Computed tomography of abdomen and pelvis without contrast C. Magnetic resonance imaging of abdomen and pelvis without contrast D. Transvaginal pelvic ultrasound

D. Transvaginal pelvic ultrasound Ectopic Pregnancy Risk factors include prior ectopic, PID, tubal surgery, IUD Symptoms include abdominal pain, pelvic pain, amenorrhea, or vaginal bleeding Labs will show positive pregnancy test and lower than expected serum beta-hCG levels Diagnosis is made by ultrasound Gestational sac with a yolk sac or embryo outside of the uterine cavity Free fluid with debris is suggestive to ruptured ectopic Most commonly located in a fallopian tube Treatment is methotrexate or surgery Abdominal X-ray (A) would not help rule out most of the differential diagnoses of the patient in the vignette, except perhaps a kidney calculus. X-ray is also not advised for patients who may be pregnant. Computed tomography of abdomen and pelvis without contrast (B) would be a good study to order if the primary differential were appendicitis. However, this patient presents with signs of a possible ectopic pregnancy, so pregnancy must be ruled out first. The fastest way to do this is via transvaginal ultrasound. Magnetic resonance imaging of abdomen and pelvis without contrast (C) is costly, time-consuming, not always available, and unnecessary in this scenario.

A 26-year-old woman presents to the emergency department with complaints of lower abdominal pain and vaginal discharge for the past 5 days. She also reports chills and malaise. Vital signs are T of 100.3°F, HR of 105 bpm, BP of 145/94 mm Hg, and RR of 20/min. On physical exam, she has tenderness to palpation in the left lower quadrant without guarding or rebound. Pelvic exam reveals purulent discharge from the cervical os, cervical motion tenderness, and left-sided adnexal tenderness. Transvaginal ultrasound shows an inflammatory multilocular mass in the left adnexal area. Which of the following is the most likely diagnosis? A. Acute diverticulitis B. Ruptured ovarian cyst C. Ruptured tubo-ovarian abscess D. Tubo-ovarian abscess without rupture

D. Tubo-ovarian abscess without rupture Tubo-Ovarian Abscess History of pelvic inflammatory disease (PID) Lower abdominal pain, fever, vaginal discharge PE will show unilateral adnexal tenderness Diagnosis is made by ultrasound Most commonly caused by a complication of pelvic inflammatory disease Treatment is intravenous antibiotics, surgical drainage, or both Acute diverticulitis (A) is due to inflammation of diverticula, which typically occurs around the sigmoid colon. The classic presentation is left lower quadrant abdominal pain, and some patients have changes in bowel movements, such as constipation or diarrhea. It occurs most often in older patients and does not cause vaginal discharge or purulent cervical drainage. A ruptured ovarian cyst (B) classically presents with acute unilateral pelvic pain in reproductive-aged women. The pain often starts during intercourse or physically strenuous activity. Transvaginal ultrasound often shows a cyst with surrounding free fluid. The systemic symptoms and vaginal discharge make TOA the most likely diagnosis in this patient. A ruptured tubo-ovarian abscess (C) is actually a misnomer since the pathologic process is more consistent with a leak than a rupture. Ruptured TOAs present with signs of an acute abdomen (involuntary guarding and rebound tenderness), and some patients have hemodynamic instability. The patient in the vignette did not have signs of an acute abdomen.

A 27-year-old G1P0 woman at 12 weeks gestation presents to her obstetrician for a routine prenatal visit. Ultrasound confirms the presence of a twin pregnancy. Which of the following is the most likely complication of multiple gestation? A. Large for gestational age infants B. Maternal breast cancer C. Post-term delivery D. Twin-twin transfusion syndrome

D. Twin-twin transfusion syndrome Large for gestational age infants (A) are associated with gestational diabetes, post-term delivery, maternal age > 35 years old, maternal obesity, excessive weight gain during pregnancy, multiparity, and male infants. Multiple gestation is associated with an increased risk of small for gestational age infants. Twin births are associated with a decreased risk of maternal breast cancer (B). Multiple gestation is associated with preterm delivery, not post-term delivery (C).

A 28-year-old woman presents to the clinic due to mastalgia for the past 2 months. She reports her pain increases with the onset of her menstrual cycle. Physical exam demonstrates a smooth, firm, discrete mobile mass. Which of the following diagnostic studies is best indicated at this time? A. Core-needle biopsy B. Diagnostic mammography C. Fine-needle aspiration D. Ultrasonography

D. Ultrasonography Fibrocystic Breast Changes Risk factors: women 30−50 years old Sx: intermittent breast pain and tenderness that peak before each menstruation Ultrasound may show dense, prominent, fibroglandular tissue with cysts but no discernible mass Most commonly caused by fluctuating hormone levels during menstrual cycles Treatment is well-fitting supportive bras, applying heat to the breasts, or over-the-counter pain relievers Most common lesion of the breast Fibrocystic changes are generally benign and do not increase risk for breast cancer Core-needle biopsy (A) and fine-needle aspiration (C) are not indicated in the initial evaluation of a patient with signs and symptoms of fibrocystic breast disease. These tests are used to definitively diagnose suspicious lesions. Diagnostic mammography (B) can be used as an alternative to ultrasound imaging, but it is less accurate in detecting microcystic changes. Based on the patient's young age and symptoms consistent with a breast cyst, ultrasound is more useful in diagnosing this condition than diagnostic mammography.

A 55-year-old postmenopausal woman presents to the clinic with vaginal spotting. She says she has not had any vaginal bleeding since her last period 4 years ago. Which of the following is a risk factor for the most concerning diagnosis? A. Combined hormonal contraception use B. Late menarche C. Smoking cigarettes D. Unopposed estrogen therapy

D. Unopposed estrogen therapy Combined hormonal contraception use (A) has reduced the risk of endometrial cancer by 30-40% in studies. The progestin in combined hormonal contraceptives reduces endometrial proliferation. In addition, progestin-only contraceptives (depot medroxyprogesterone acetate, progestin-implants, and progestin-releasing intrauterine devices) provide even more protection than combined hormonal contraceptives. Late menarche (B) is incorrect. Early menarche and late menopause are risk factors for endometrial cancer because more menstrual cycles expose the uterus to more estrogen over time. Smoking cigarettes (C) is actually a protective factor for endometrial cancer in postmenopausal women. It stimulates the hepatic metabolism of estrogen, which reduces the risk of endometrial cancer. However, the health risks of smoking cigarettes far outweigh any benefit.

What are the causes of fetal tachycardia?

Fetal movement, maternal factors (anemia, hyperthyroidism, hypoxia, dehydration, fever or sepsis, and anxiety), fetal factors (dysrhythmias), and placental abruption.

What are some causes of fetal hypoxia?

Maternal hypertensive disease, maternal heart disease, maternal infection, placental insufficiency, fetal growth restriction, and umbilical cord prolapse.

Why do hyperthyroidism and theca lutein cysts occur during gestational trophoblastic disease?

The alpha subunit of human chorionic gonadotropin closely mimics thyroid-stimulating hormone and luteinizing hormone.


Related study sets

AP Euro Essential Vocabulary 3.3

View Set

Elasticity and Its Application (Ch05)

View Set

microbiology exam 2 review pt. 1

View Set

politics of developing areas midterm

View Set

U.S. Government Chapter 3 REVIEW

View Set

Chapter 1: The Financial Statements - Homework

View Set

TLB-Chapter 24: Asepsis and Infection Control

View Set